Monthly Prestorming May 2023 - WWW - Iasparliament

You might also like

Download as pdf or txt
Download as pdf or txt
You are on page 1of 66

www.shankariasacademy.

com 2
www.iasparliament.com

CHENNAI |SALEM| MADURAI | COIMBATORE DELHI | BANGALORE | T HIRUVANANTHAPURAM


www.shankariasacademy.com 2
www.iasparliament.com

INDEX

2
01-05-2023................................................................................................ 3
03-05-2023................................................................................................ 6
04-05-2023................................................................................................ 9
05-05-2023.............................................................................................. 11
06-05-2023.............................................................................................. 14
08-05-2023.............................................................................................. 17
09-05-2023.............................................................................................. 20
10-05-2023.............................................................................................. 22
11-05-2023.............................................................................................. 25
12-05-2023.............................................................................................. 28
13-05-2023.............................................................................................. 31
15-05-2023.............................................................................................. 34
16-05-2023.............................................................................................. 37
17-05-2023.............................................................................................. 40
18-05-2023.............................................................................................. 42
19-05-2023.............................................................................................. 45
20-05-2023.............................................................................................. 47
21-05-2023.............................................................................................. 50
23-05-2023.............................................................................................. 53
24-05-2023.............................................................................................. 55
25-05-2023.............................................................................................. 58
26-05-2023.............................................................................................. 61
30-05-2023.............................................................................................. 63

CHENNAI |SALEM| MADURAI | COIMBATORE DELHI | BANGALORE | T HIRUVANANTHAPURAM


www.shankariasacademy.com 3
www.iasparliament.com

MONTHLY PRESTORMING

May 2023

01-05-2023 3. Paliy ans, Mala Pandarams and Ulladans are


some of the tribes found in the reserve.
1 ) Consider the following statements with respect to
States Reorganization Act, 1 956 Which of the statement(s) giv en above is/are correct?

1. It is based on the recommendation of JV P a. 1 and 2 only


committee on linguistic basis.
b. 2 and 3 only
2. Andhra Pradesh is the first state to be separated
after the commencement of constitution. c. 2 only

3. In 1 960, the bilingual state of Bombay was d. 1 , 2 and 3


div ided into Maharashtra and Gujarat.

Which of the statement(s) giv en above is/are correct?


4) Consider the following statements with respect to
a. 1 only Laundromat Countries

b. 1 and 2 only 1. These countries buy Russian oil and sell


processed products to European countries, thus
c. 3 only by -passing European sanctions against Russia.

d. 1 , 2 and 3 2. The laundromat countries are China, India,


Turkey , the UAE and Singapore.

Which of the statement(s) giv en above is/are correct?


2) Consider the following statements with respect to
Lithium Reserv es and Lithium Production a. 1 only

1. Lithium is a lightweight metal used in the b. 2 only


cathodes of lithium-ion batteries, which power
electric v ehicles. c. Both 1 and 2

2. Australia is the largest producer of lithium d. Neither 1 nor 2


followed by Chile and China.

3. Salar de Atacama, Chile has the largest


reserves. 5) Consider the following statements with respect to
H5N1 V irus
Which of the statement(s) giv en above is/are correct?
1. It is a Highly Pathogenic Av ian Influenza
a. 1 only (HPAI)-A in Birds and Other Animals.

b. 2 and 3 only 2. The v irus does not infect humans easily , and
spread from person to person appears to be
c. 1 and 2 only unusual.

d. 1 , 2 and 3 3. There is currently no v accine av ailable to treat


H5N1 in humans.

Which of the statement(s) giv en above is/are correct?


3) Consider the following statements with respect to
Periy ar Tiger Reserve a. 1 only

1. It spreads only in the Southern state of Tamil b. 1 and 2 only


Nadu.
c. 2 and 3 only
2. It consists of semi- ev ergreen and moist
deciduous forests. d. 1 , 2 and 3

CHENNAI |SALEM| MADURAI | COIMBATORE DELHI | BANGALORE | T HIRUVANANTHAPURAM


www.shankariasacademy.com 4
www.iasparliament.com

Answers Hary ana, Chandigarh and Him achal Pradesh

1. c  In 1 966, the State of Punjab was bifurcated11 to


create Hary ana, the 1 7 th state of the Indian
Gujarat and Maharashtra born out of bilingual Union, and the union territory of Chandigarh.
Bombay State celebrate their states day
Manipur, T ripura and Meghalaya
States Reorganization Act, 1956
 Manipur 1 9th, Tripura 20th and Meghalaya
 Andhra Pradesh is the first state to be created 21 st state created in 1 97 2.
by separating the Telugu speaking areas from
the Madras state because of the death of Potti Sikkim
Sriramulu.
 The 36th Constitutional Amendment Act (1975)
 The creation of Andhra state intensified the was enacted to make Sikkim a full-fledged state
demand from other regions for creation of of the Indian Union (the 22nd state).
states on linguistic basis.
Mizoram , Arunachal Pradesh and Goa
 So Fazl Ali commission is formed to re-examine
the recommendations and submitted its report  In 1 987 , Mizoram, Arunachal Pradesh and Goa
based on linguistic states. came into being as the 23rd, 24th and 25th
states of the Indian Union respectively.
 But it rejected the theory of ‘one language-one
state’. Chhattisgarh, Uttarakhand and Jharkhand

 By the states reorganization act the 4 fold  In 2000, 3 more new States of Chhattisgarh,
classification of states is abolished and 14 states Uttarakhand and Jharkhand were created as
and 6 union territories were created on the 26th, 27 th and 28th states of the Indian
Nov ember 1 , 1 956. Union, respectively.

Maharashtra and Gujarat T elangana

 In 1 960, the bilingual state of Bombay was  In 201 4, the new state of Telangana came into
div ided into two separate states–Maharashtra ex istence as the 29th state of the Indian Union.
for Marathi speaking people and Gujarat for
Gujarati speaking people. Jam m u & Kashm ir and Ladakh

 Gujarat was established as the 1 5th state of the  The union territory of Jammu and Kashmir is
Indian Union. created by abolishing the special status by a
presidential order under the Constitution
Dadra and Nagar Hav eli (Application to Jammu and Kashmir) Order,
201 9.
 It was conv erted into a union territory of India
by the 1 0th Constitutional 2. d

 Amendment Act, 1 961. Chile’s President Gabriel Boric announced plans for the
state to take a majority stake in the country’s lithium
Goa, Dam an and Diu industry

 Goa was conferred a statehood. Lithium

 Consequently, Daman and Diu was made a  Lithium is a lightweight metal used in the
separate union territory. cathodes of lithium-ion batteries, which power
Electric V ehicles (EV ).
Puducherry
 The need for lithium has increased significantly
 1 962 - It was made a union territory by the 14th due to the growing demand for EV s.
Constitutional Amendment Act.
 Howev er, lithium is also used in the batteries of
Nagaland laptops and cell phones, as well as in the glass
and ceramics industry.
 In 1 963, the State of Nagaland was formed10 by
taking the Naga Hills and Tuensang area out of  Lithium is also used to make lubricant greases
the state of Assam. for the transport, steel, and av iation industries,
along with other lesser-known uses.

CHENNAI |SALEM| MADURAI | COIMBATORE DELHI | BANGALORE | T HIRUVANANTHAPURAM


www.shankariasacademy.com 5
www.iasparliament.com

Lithium Production and Reserves 4. c

 Lithium from Australia comes from ore mining, Among the laundromat countries, India remained the
while in Chile and Argentina lithium comes highest global consumer of seaborne Russian crude in
from salt deserts, so-called salars. April.

 The 3 largest producers of lithium are Australia, Laundromat Countries


Chile and China.
 These are the countries that buy Russian oil and
 Chile has the Salar de Atacama, one of the sell processed products to European countries,
world’s richest lithium brine deposits. thus ev ading European sanctions against
Russia.
 The demand for lithium is ex pected to reach 1.5
million tonnes of lithium carbonate equivalent  The laundromat countries are China, India,
by 2025 and over 3 million tonnes by 2030. Turkey , the UAE and Singapore.

 The global market for the alkali metal lithium is  These countries are refining larger v olumes of
growing rapidly. imported Russian crude to then export refined
products to sanction imposing countries.
3. b
 This is currently a legal way of ex porting oil
Periy ar T iger Reserve products to countries that are imposing
sanctions on Russia.
 The Periy ar Tiger Reserv e (PTR) in Thekkady
located in the southern region of the Western  These countries ex port nearly 3.8 million
Ghats in Tamil Nadu and Kerala. tonnes of oil products to price cap coalition
countries, which include the EU, G-7 countries,
 It gets its name from the Riv er Periy ar which Australia and Japan.
has its origin deep inside the reserve.
Centre for Research on Energy and Clean Air
 It was declared as a Sanctuary in 1 950 and (CREA) report
declared as Tiger Reserve in 1 97 8.
 The CREA report said the most oil products
 Habitat - The terrain is Hilly and undulating. were being ex ported from two ports in Gujarat:

 Riv ers - Two major riv ers namely Periyar and  The Sikka port that serv ices the Reliance-
Pamba drain the area. owned Jamnagar refinery, and

 Mullai Periy ar Dam is located within the PTR.  The V adinar port that ships oil products from
Nay ara energies.
 The v egetation comprises of Tropical evergreen
forests, semi- ev ergreen forests, Moist  RIL’s Jamnagar refinery, is the biggest oil
deciduous Forests, Transitional fringe ever product export port to the price -cap coalition
green forests, grass lands and eucalyptus countries.
plantations.
 It is also the largest importing port in the world
 Fauna - In addition to Tiger, Periy ar is an of seaborne crude oil from Russia.
important elephant habitat.
5. d
 Gaur, sambar deer, leopard, sloth bear, wild
dog, Lion tailed monkey, Nilgiri langur, barking Uncontrolled spread of H5N1 virus leading to the mass
deer and Nilgiri tahr are found here. extinction of several species of endangered birds.

 The major four species of primates are also H5N1 Virus


found at Periy ar - the rare lion-tailed macaque,
the Nilgiri Langur, Gee's Golde n Langur,  Av ian influenza A v iruses are designated as
Common Langur and Bonnet Macaque. highly pathogenic avian influenza (HPAI) or
low pathogenicity av ian influenza (LPAI) based
 T ribes - Paliy ans, Mannans, Malay arayans, on molecular characteristics of the v irus.
Mala Pandarams, Uralis and Ulladans.
 H5N1 is a Highly Pathogenic Avian Influenza A
(H5N1 ) in Birds and Other Animals.

 It is a reassortant virus with genes from Asian


HPAI H5 v iruses and low pathogenic North
American v iruses.

CHENNAI |SALEM| MADURAI | COIMBATORE DELHI | BANGALORE | T HIRUVANANTHAPURAM


www.shankariasacademy.com 6
www.iasparliament.com

 Reassortm ent –It occurs when the genes 3) Consider the following statements with respect to
from two different v iruses mix to create a new National Manufacturing Innovation Survey, 2021-22
v irus.
1. It is a joint initiativ e of Department of Science
 World Health Organization (WHO) has and Technology (DST) and the United Nations
assessed the risk of H5N1 to humans to be low. Industrial Dev elopment Organization
(UNIDO).
 United Nations Food and Agriculture
Organization considered 6 countries to be 2. According to the survey, MSMEs are the most
endemic for Asian HPAI H5N1 v irus in poultry innov ative sectors in manufacturing sectors of
(Bangladesh, China, Egy pt, India, Indonesia, India.
and V ietnam).
3. According to the surv ey, Karnataka is the most
innov ative state in India.

Which of the statement(s) giv en above is/are correct?


03-05-2023
1 ) Consider the following statements with respect to Red a. 1 and 2 only
Pandas
b. 2 and 3 only
1. They liv e in cold, mountainous mixed
deciduous and coniferous forests of the eastern c. 1 and 3 only
Himalay as.
d. 1 , 2 and 3
2. It is listed as critically endangered in IUCN’s
Red data list.

3. India is habitat for both the Himalay an Red 4) Consider the following statements
Panda and the Chinese Red panda.
1. The Constitution of India specifies the official
Which of the statement(s) giv en above is/are correct? languages of different states.

a. 1 only 2. The legislature of a state may adopt one or more


of the languages in use in the state or Hindi as
b. 1 and 3 only the official Language of the state.

c. 2 and 3 only Which of the statement(s) giv en above is/are correct?

d. 1 , 2 and 3 a. 1 only

b. 2 only

2) Consider the following statements with respect to c. Both 1 and 2


V olcanoes
d. Neither 1 nor 2
1. Magma is a melted rock mix ed with gas and
mineral cry stals accumulates in an
underground chamber beneath the v olcano.
5) Consider the following statements with respect to
2. Stratov olcano hav e relatively steep sides and Conv ectional Rainfall
are more cone-shaped than shield v olcanoes.
1. Conv ectional rainfall occurs when the energy of
3. Mauna Loa is the world’s largest active v olcano. the sun heats the earth’s surface and the
ev aporation results in rainfall during the
Which of the statement(s) giv en above is/are correct? summer season.

a. 1 only 2. This ty pe of rainfall is widespread in tropical


areas.
b. 1 and 2 only
Which of the statement(s) giv en above is/are correct?
c. 2 and 3 only
a. 1 only
d. 1 , 2 and 3
b. 2 only

c. Both 1 and 2

CHENNAI |SALEM| MADURAI | COIMBATORE DELHI | BANGALORE | T HIRUVANANTHAPURAM


www.shankariasacademy.com 7
www.iasparliament.com

d. Neither 1 nor 2  These animals are hunted for their lucrative red
skin and meat.

 They are also captured liv e and traded as pets.


Answers
1. b 2. d

Red Panda Mauna Loa, the w orld’s largest active volcano erupted
on the island of Haw aii.
 Habitat and most of the corridors of Red
Panda are outside the Protected Area network. Mechanism behind Volcanic Eruption

 They liv e in chilly , cold, mountainous mixed  The center of the Earth is called the core, the
deciduous and coniferous forests of the eastern nex t lay er up is the mantle, the outermost layer
Himalay as. is the crust.

 Altitude - Ranging from 2,200 metres and  Magma is a melted rock mix ed with gas and
5,000 m. mineral cry stals accumulates in an
underground chamber beneath the v olcano.
 T emperature - They prefer a v iable
temperature range of 1 0 ºC to 25 ºC.  The magma forms when a hot mantle plume
that melts the rock.
Spotted in National Parks
 The v olcano is an opening that lets magma out
 Singalila National Park – Darjeeling. onto the surface of the Earth.

 Neora V alley national park - West Bengal.  Once released from the v olcano, the magma is
called lav a.
 Kangchenjunga National Park - Sikkim.
 Earthquakes - If more magma was making its
 Ex -situ site of Padmaja Naidu Himalayan way upward, because pressure from rising
Zoological Park (PNHZP) in West Bengal. magma can ex pand the sides of a v olcano and
cause rocks to shift and break causes
 PNHZP is the only zoo in India designated for earthquakes.
the conservation and breeding of Red Pandas in
captiv ity. T y pes of v olcanoes

 Transboundary conservation is essential to save  Shield v olcano – It built up ov er thousands


endangered Red Pandas. of y ears through lav a eruptions and its sides
slope gently downward in all directions.
T y pes of Red Pandas
E.x - Mauna Loa
 T he Him alayan Red Panda – Found in
Sikkim, Darjeeling, West Bengal, Nepal,  Com posite volcano –It is also known as a
Bhutan and Southern Tibet. stratovolcano, with steep sides that come to a
point at the top, like an upside-down sugar
 T he Chinese Red Panda - Southeastern cone.
Tibet, Northern My anmar and the Sichuan and
Y unan provinces of China. E.x - Mount Semeru

 Y arlung Zangbo River acts as the actual


geographical barrier between the distributions
of these two species.

 The zy gomatic (cheekbone) breadth and skull


size of the Chinese Red Panda is larger than the
Himalay an Red Panda.

 The facial colour of the Chinese Red Panda is


redder than the Himalay an Red Panda.

 In the Chinese Red Panda, tail rings are more


prominent, with dark rings being darker red
and the pale rings being whitish than the
Himalay an Red Panda.

CHENNAI |SALEM| MADURAI | COIMBATORE DELHI | BANGALORE | T HIRUVANANTHAPURAM


www.shankariasacademy.com 8
www.iasparliament.com

3. c 4. b

Karnataka ranked the most innovative state by the Union Law Minister Kiren Rijiju said regional
National Manufacturing Innovation Survey languages should be used in courts in every State.

National Manufacturing Innovation Survey, Prom otion of Regional Languages


2021-22
 The Constitution of India doesn’t specifies the
 It is a joint initiativ e of Department of Science official languages of different states.
and Technology (DST) and the United Nations
Industrial Dev elopment Organization  The legislature of a state may adopt one or more
(UNIDO). of the languages in use in the state or Hindi as
the official Language of the state.
 Aim - It aims to understand the current
innov ation activ ities of the manufacturing  Until that English is the official Language of
economy in India. that state.

 It is a two-pronged survey that ex amined the  T he Official Languages Act (1963) lays
innov ation processes, outcomes, and barriers in down that English should be the
manufacturing firms, and also studied the communication language between the Union
innov ation ecosystem that affects innov ation and the states for non-hindi states.
outcomes in these firms.
 When the President is satisfied that a
 The ex ercise is a follow-up of the DST’s first substantial portion of the population of a state
National Innovation Surv ey held in 2011. desired the use of any language spoken by them,
he may recognized that as the official language
Findings of the Surv ey of that state.

 Degree of Innovation among  This provision aims at protecting the linguistic


Manufacturing Firm s minorities in states.

 Karnataka is the most innov ative state overall  Citizens can use any language other than the
followed by Dadra and Nagar Hav eli, Daman eighth schedule language for the redressal
and Diu (DNH&DD), Telangana, and Tamil griev ances in the court.
Nadu.
5. c
 Highest share of Innovative firm s -
Telangana, Karnataka, and Tamil Nadu Conv ectional Rainfall

 Lowest share of Innovative firm s -  Conv ectional rainfall occurs when the energy of
Odisha, Bihar, and Jharkhand. the sun (or insolation) heats the earth’s surface
and causes water to ev aporate changing to
 Nearly 3/4th of the 8,000 -odd firms neither water v apor.
made any innov ative product nor business
process innovation during the survey period of  This warm, moist air then rises, and as it rises,
financial y ears 2017-2020. it cools.

 Most of them are micro, small, and medium  The air reaches a point called the condensation
enterprises (MSME). lev el where it has cooled to such an ex tent that
the water v apor condenses and turns back to a
 Howev er, nearly 80% of the firms that did liquid form.
reported significant gains such as ex panding
markets and reducing production costs.  This process of condensation high in the
atmosphere leads to the development of clouds.
 Barriers to Innovation - Lack of internal
funds, high innov ation costs, and lack of  As the clouds continue to grow the weight of the
financing from ex ternal sources. water droplets can ev entually lead to
precipitation.
 Gujarat and DNH&DD reported the highest
frequencies of barriers to innov ation, despite  This ty pe of rainfall is widespread in tropical
being among India’s most industrialised States. areas (between the Tropic of Cancer and the
Tropic of Capricorn) and in areas such as
South-east England during warm sun.

CHENNAI |SALEM| MADURAI | COIMBATORE DELHI | BANGALORE | T HIRUVANANTHAPURAM


www.shankariasacademy.com 9
www.iasparliament.com

04-05-2023 4) Consider the following statements with respect to


Leopards
1 ) Y anomami, are a group of indigenous people
inhabited in the regions of? 1. It is the smallest of the big cats, and known for
its ability to adapt in a v ariety of habitats.
a. V enezuela and Brazil
2. Indian Leopard is listed as endangered in the
b. Colombia and V enezuela IUCN’s red data list.
c. France and Spain Which of the statement(s) giv en above is/are correct?
d. Brazil and Boliv ia a. 1 only

b. 2 only
2) Consider the following statements with respect to c. Both 1 and 2
World Press Freedom Index
d. Neither 1 nor 2
1. It aims to compare the lev el of press freedom
enjoy ed by journalists and media in 180
countries and territories.
5) Consider the following statements with respect to
2. It is an annual ranking of countries compiled V igy an Prasar
and published by Reporters without Borders
(RSF). 1. It is an autonomous organization setup by the
department of science and technology.
3. India made a significant improv ement on the
index in 2023 compared to 2022. 2. It aims to promote scientific temper and
enhancing public understanding of science
Which of the statement(s) giv en above is/are correct? through communication.

a. 1 and 2 only Which of the statement(s) giv en above is/are correct?

b. 1 and 3 only a. 1 only

c. 2 and 3 only b. 2 only

d. 1 , 2 and 3 c. Both 1 and 2

d. Neither 1 nor 2

3) Consider the following statements with r espect to


Global Report on Food Crisis
Answers
1. It is an annual report launched by the Global
Network against Food Crisis (GNAFC). 1. a

2. It is facilitated by the Food Security President Lula-led Brazilian government has promised
Information Network (FSIN). to speed up the expulsion of illegal miners in the
Y anomami, after 5 people w ere reportedly killed.
3. More than half of the people affected by acute
hunger is located in Afghanistan, the Y anomami
Democratic Republic of the Congo, Sudan and
Y emen.  Y anomami are the South American Indians.

Which of the statement(s) giv en above is/are correct?  Language - Xiriana language

a. 1 only  Habitat - Liv e in the remote forest of the


Orinoco Riv er basin in southern V enezuela and
b. 2 and 3 only the northernmost reaches of the Amazon River
basin in northern Brazil.
c. 3 only
 The Y anomami liv e in v ine-and-leaf-thatched
d. 1 , 2 and 3 houses in palisaded v illages surrounded by
garden plots.

CHENNAI |SALEM| MADURAI | COIMBATORE DELHI | BANGALORE | T HIRUVANANTHAPURAM


www.shankariasacademy.com 10
www.iasparliament.com

 They relocate their v illages when the soil wears o Transparency, and
out or when a v illage has become too
susceptible to attack by other Yanomami. o The quality of the infrastructure that
supports the production of news and
 Occupation - They practice slash-and-burn information.
agriculture and liv e in small, scattered,
semipermanent v illages. Rankings, 2023

 They grow tobacco, a great fav ourite of  India’s ranking in the 2023 World Press
Y anomami of all ages, and cotton, an important Freedom Index has slipped to 1 61 out of 180
trade and domestic item used in the countries compared to 2022.
manufacture of string and cord for hammocks,
nets, containers, and clothing.  Top 3 positions - Norway , Ireland and
Denmark.
 They keep dogs, both for village security and for
hunting.  Bottom 3 positions - V ietnam, China and North
Korea.
 Social life – It centres on forming alliances
through trade and sharing food with other  Pakistan - 1 50
friendly groups while waging war against
hostile v illages.  Sri Lanka - 1 35

 Y anomami’s life and surv ival were threatened 3. d


by the incursions of Brazilian miners into their The Global Netw ork against Food Crisis report, 2023
territory in the Brazilian state of Roraima from
released recently.
1 990.
Global Report on Food Crisis, 2023
2. a
 It is an annual report launched by the Global
India slips in World Press Freedom Index, ranks 161
Network against Food Crises (GNAFC) - an
out of 1 80 countries in 2023.
international alliance of the United Nations, the
World Press Freedom Index European Union, gov ernmental and non-
gov ernmental agencies.
 Aim - It aims to compare the lev el of press
freedom enjoyed by journalists and media in  It is facilitated by the Food Security
1 80 countries and territories. Information Network (FSIN).

 Agency - It is an annual ranking compiled and Findings of the report


published by Reporters without Borders (RSF).
 More than half of the people were on the edge
of starv ation in Somalia.
 This qualitative analy sis is combined with
quantitative data on abuses and acts of v iolence
 Ex treme circumstances also occurred in
against journalists during the period evaluated.
Afghanistan, Burkina Faso, Haiti, Nigeria,
Param eters South Sudan and Y emen.

 Qualitative param eter - Based on the  The report also found that around 35 million
responses of “press freedom specialists to an people ex perienced emergency levels of acute
RSF questionnaire. hunger in 39 countries.

 Quantitative parameter - A count of abuses  More than half of them are located in just 4
against journalists and media outlets. countries — Afghanistan, the Democratic
Republic of the Congo, Sudan and Y emen.
 The criteria considered in this are
 Acute food insecurity - When a person’s
o Pluralism, inability to consume adequate food puts their
liv es or liv elihoods in immediate danger.
o Media Independence,
Food Security Information Network (FSIN)
o Media Env ironment and
 It is a global initiative to strengthen food and
o Self-censorship, nutrition security information sy stems for
producing reliable and accurate data to guide
o Legislativ e framework, analy sis and decision-making.

CHENNAI |SALEM| MADURAI | COIMBATORE DELHI | BANGALORE | T HIRUVANANTHAPURAM


www.shankariasacademy.com 11
www.iasparliament.com

 It promotes independent and consensus-based  Status - The International Union for


information and highlights critical data gaps. Conserv ation of Nature (IUCN) has classified
Indian Leopard as a v ulnerable species.
 It is co-sponsored by FAO, WFP and IFPRI.
5. c
4. a
An official press release of the Department of Science
A new portal dedicated to leopards was launched by and Technology (DST) announced the end of V igyan
the Cape Leopard Trust (CLT) on International Prasar.
Leopard Day.
Vigy an Prasar
Leopards
 It started functioning in 1 989.
 It is also called as panther.
 It was a Nehruv ian idea of science
 The large cat is closely related to the lion, tiger, communication and scientific awareness
and jaguar. tracing itself to the early days of independence.

 Geographic range - Whole of Africa south of  It is an autonomous organization setup by the


the Sahara, occupied parts of north and department of science and technology.
northeast Africa, and extended from Asia Minor
through Central Asia and India to China and  Aim - It aims to promote scientific temper and
Manchuria. enhancing public understanding of science
through communication.
 9 subspecies of the leopard hav e been
recognized, and they are distributed across  This is achieved through sev eral strategically
Africa and Asia. important two – way stakeholder specific
approaches.
 By 201 9 the species had lost up to 7 5 percent of
its former range.  It aims to communicate about principles and
practice of science and technology and
 The leopard is a solitary animal of the bush and implications for dev elopment and quality of
forest and is mainly nocturnal in habit, life.
although it sometimes basks in the sun.
 Science popularization, therefore, serves as a
 Habitat - Leopards inhabit a v ariety of terrain. robust knowledge led tool to fulfill three
mutually reinforcing public policy objectives.
 They are most populous in mesic woodlands,
grassland sav annas, and forests.  In addition, the V igy an Prasar management
also opened a rev olving door between V igyan
 They also occupy mountainous, scrub, and Prasar and ‘V igy an Bharti’ aka V IBHA, an
desert habitats. affiliate of the Rashtriy a Sway amsevak Sangh.

Indian leopard  The rise of V IBHA has coincided with the


decline of V igyan Prasar in the past 5 y ears.
 It is distributed in India, Nepal, Bhutan and
parts of Pakistan.

 It inhabits tropical rainforests, dry deciduous 05-05-2023


forests, temperate forests and northern
coniferous forests. 1 ) Consider the following statements with respect to
Self-Reliant India (SRI) Fund
 It does not occur in the mangrove forests of the
Sundarbans. 1. Self-Reliant India (SRI) is a SEBI
registered Category II Alternative Inv estment
 It is thought that the Indus Riv er in the west Fund.
and the Himalay as in the north form
2. It aims to make capital inv estments in micro,
topographical barriers to the dispersal of this
subspecies. small and medium enterprises (MSMEs).

3. It is managed by the SBICAP V entures (SV L).


 In the east, the Ganges Delta and the lower
course of the Brahmaputra Riv er are thought to Which of the statement(s) giv en above is/are correct?
form natural barriers to the range of the Indo-
chinese leopard. a. 1 only

CHENNAI |SALEM| MADURAI | COIMBATORE DELHI | BANGALORE | T HIRUVANANTHAPURAM


www.shankariasacademy.com 12
www.iasparliament.com

b. 1 and 2 only d. Sea Lev el Rise

c. 2 and 3 only

d. 1 , 2 and 3 5) Consider the following statements with respect to


Washington Declaration adopted recently

1. It is nuclear deterrence strategy adopted jointly


2) Consider the following statements with respect to by the U.S. and the South Korea.
Meitei
2. It mandates the U.S. as the only sole authority
1. Meitei are the dominant people of the to use the nuclear arsenal of the U.S. in the
northeastern state of Meghalaya. ev ent of a nuclear confrontation.

2. They speak Tibeto -Burman language. Which of the statement(s) giv en above is/are correct?

3. Manipur gov ernment has recently granted a. 1 only


them the Scheduled Tribes (ST) status.
b. 2 only
Which of the statement(s) giv en above is/are correct?
c. Both 1 and 2
a. 1 and 2 only
d. Neither 1 nor 2
b. 2 and 3 only

c. 2 only
Answers
d. 1 , 2 and 3
1. d

Recently, Ministry of MSME launched an outreach


3) Consider the following statements with respect to the programme under the Self Reliant India (SRI) Fund at
Protection of Women against Sex ual Harassment at Dimapur, Nagaland.
Workplace (PoSH) Act, 2013
SRI (Self-Reliant India) Fund
1. According to this act, ev ery employer must
constitute an Internal Complaints Committee  It is a SEBI registered Category II Alternative
(ICC) in their organization to address Inv estment Fund.
complaints of sexual harassment.
 Aim - It aims to make equity inv estments in
2. It is enacted based on the V ishaka guidelines. micro, small and medium enterprises (MSMEs)
to address the current capital gap and catalyze
3. Under the A ct, the complaint must be made the flow of capital in the sector.
within 3 months from the date of the incident.
 The anchor inv estor of the Fund is the
Which of the statement(s) giv en above is/are correct? Gov ernment of India, through the Ministry of
Micro, Sm all and Medium Enterprises.
a. 1 and 2 only
 SRI Fund has been sponsored by The
b. 2 only National Sm all Industries Corporation
Ltd. (a mini-Ratna Corporation of the
c. 2 and 3 only Gov ernment of India under the Ministry of
Micro, Small and Medium Enterprises).
d. 1 , 2 and 3
 It is setup under its wholly owned
subsidiary, NSIC Venture Capital Fund
4) Planetary boundaries are a concept describing limits Lim ited (NVFCL).
within which humanity can continue to dev elop and
thriv e for generations to come. Which of the following is  NV CFL has appointed SBICAP VenturesLtd
not among the planetary boundaries? (SVL), an Asset Management Company, as the
Inv estment Manager of the Fund.
a. Climate Change
Objectives
b. Nitrogen and Phosphorus (geochemical) Cy cles
 Helping MSME businesses grow faster so as to
c. Ocean Acidification energize the economy and create employment
opportunities;

CHENNAI |SALEM| MADURAI | COIMBATORE DELHI | BANGALORE | T HIRUVANANTHAPURAM


www.shankariasacademy.com 13
www.iasparliament.com

 Supporting enterprises which hav e the 3. d


potential to graduate bey ond the MSME
bracket and become National / International An investigation revealed that more than half of India’s
champions; and national sports federations do not have an Internal
Complaints Committee (ICC)
 Supporting MSMEs which help making India
self-reliant by producing relevant technologies, Protection of Wom en against Sexual
goods and serv ices. Harassm ent at Workplace (PoSH) Act, 2013

2. c  The Sex ual Harassment of Women at


Workplace (Prev ention, Prohibition and
Meitei community demanded to be included in the list Redressal) Act, commonly known as the PoSH
of the state’s Scheduled Tribes of Manipur. Act, was passed in 201 3.

Meitei  It is enacted based on the ex tension of ex isting


v ishaka guidelines.
 Meitei also called Manipuri is the dominant
population of Manipur in northeastern India.  Ministry – Ministry of women and child
dev elopment.
 Ethnicity - The area was once
inhabited entirely by peoples resembling such  It defined sex ual harassment, lay down the
hill tribes as the Naga and the Mizo. procedures for complaint and inquiry, and the
action to be taken in cases of sexual
 Intermarriage and the political dominance of harassment.
the strongest tribes led to a gradual merging of
ethnic groups finally lead to the formation of  The PoSH Act subsequently mandated that
the Meitei. ev ery employer must constitute an Internal
Complaints Committee (ICC) at each office or
 They are div ided into clans, the members of branch that had 1 0 or more employees.
which do not intermarry.
Key prov isions of the POSH Act, 2013
 Language - They speak a Tibeto -Burman
language.  Ev ery employer is required to display a notice
in the organisation prov iding details of the
 They are culturally different from the protection giv en to female employ ees against
surrounding hill tribes by following Hindu sex ual harassment.
customs.
 Employ ers must constitute an Internal
 Before their conv ersion to Hinduism, they ate Complaints Committee (ICC) in their
meat, sacrificed cattle, and practiced organisation to address complaints of sexual
headhunting. harassment.

 Now they abstain from meat (though they eat  A woman must head the ICC; at least half of its
fish), do not drink alcohol, observe rigid rules members should be women.
against ritual, pollution, and revere the cow.
 Employ ers must take steps to prev ent sexual
 They claim high-caste status. harassment and ensure that the victims are not
v ictimised or discriminated against.
 Worship - The worship of Hindu gods, with
special dev otion to Krishna, has not prohibited  Employ ers must provide necessary support and
the worship of many pre- assistance to the complainant and make
Hindu indigenous deities and spirits. arrangements for her work in case she has to be
transferred.
 Occupation -Rice cultiv ation on irrigated
fields is the basis of their economy.  Procedure for com plaint - It is not
compulsory for the aggriev ed v ictim to file a
 They are keen horse breeders, and polo is a complaint for the ICC to act.
national game.
 If the woman cannot complain because of
 Field Hockey , boat races, theatrical phy sical or mental incapacity or death or
performances, and dancing well known otherwise, her legal heir may do so.
throughout India as the Manipuris sty le are
other pastimes.  Under the Act, the complaint must be made
within 3 months from the date of the incident.

CHENNAI |SALEM| MADURAI | COIMBATORE DELHI | BANGALORE | T HIRUVANANTHAPURAM


www.shankariasacademy.com 14
www.iasparliament.com

 Howev er, the ICC can ex tend the time limit 5. c


upon its satisfaction.
South Korean President arrived in the U.S. to
 The ICC may either forward the v ictim’s commemorate the 7 0th anniversary of U.S. -South
complaint to the police, or it can start an inquiry Korea bilateral relations and signed Washington
that has to be completed within 90 days. Declaration as a nuclear deterrence strategy.

4. d Washington Declaration

The 2nd edition of the Global Land Outlook report has Prov isions of the declaration
reported that humans have breached 4 out of 9
planetary boundaries.  According to the declaration, an American
nuclear ballistic submarine would be deployed
Global Land Outlook report in the Korean peninsula.

 The report is prepared by the United Nations  A nuclear consultative group would be formed
Conv ention to Combat Desertification to formulate principles of joint response tactics.
(UNCCD).
 South Korea would receive Intel from the U.S.
 The report envisaged to draw attention for the regarding nuclear advancements.
depletion of planetary boundaries and the need
to restore the world’s land.  The U.S. will strengthen South Korea’s nuclear
deterrence capabilities through joint military
Planetary Boundaries training programs and an annual
intergovernmental simulation.
 Planetary boundaries are a framework to
describe limits to the impacts of human  The declaration reaffirmed the non-
activ ities on the Earth sy stem. proliferation Treaty implying that South Korea
would not v enture into the creation of its own
 If these boundaries are crossed, it would lead to independent nuclear capabilities.
abrupt or irrev ersible planetary changes that
would hav e a large-scale impact.  It would instead focus on deterrence measures
through an alliance-based approach.
 The 9 planetary boundaries are:
 It also mandates the U.S. President as the only
o Biodiv ersity loss ‘sole authority’ to use the nuclear arsenal of the
U.S. in the ev ent of a nuclear confrontation.
o Land-use change
 While the ex istence of the agreement is based
o Climate change on the security needs of South Korea, the policy
reflects big power politics where the interests of
o Nitrogen and phosphorus
the larger power (U.S.) takes precedence.
(geochemical) cycles
 The Washington Declaration adv ocates for
o Freshwater use
nuclear deterrence policy in the region, aiming
o Ocean acidification to balance power dy namics against North
Korea.
o Chemical pollution

o Atmospheric loading
06-05-2023
o Ozone depletion
1 ) Consider the following statements with respect to
Genetically Modified (GM) and Gene Editing (GE)
 Of these, climate change, biodiv ersity loss,
techniques:
land-use change, and geochemical cycles have
already been exceeded. 1. In GM the ex isting genetic material is modified
in way s that can y ield beneficial outcomes.
 These breaches are directly linked to human-
induced desertification, land degradation, and 2. In Gene editing methods the genetic material
drought. deriv ed from other organism is introduced to
the parent organism.

Which of the abov e statement(s) is/are incorrect?

a. 1 only

CHENNAI |SALEM| MADURAI | COIMBATORE DELHI | BANGALORE | T HIRUVANANTHAPURAM


www.shankariasacademy.com 15
www.iasparliament.com

b. 2 only 3. The Kremlin is part of the UNESCO World


Heritage Sites.
c. Both 1 and 2
Which of the abov e statement(s) is/are correct?
d. Neither 1 nor 2
a. 1 and 2 only

b. 2 and 3 only
2) Consider the following statements with respect to the
Petersberg Climate Dialogue: c. 1 and 3 only

1. It is the informal forum for focusing both on d. 1 , 2 and 3


international climate negotiations and the
adv ancement of climate action.

2. It works under the aegis of the Global 5) Consider the following statements with respect to Co-
Env ironment Facility. lending:

3. 2023 is the y ear for the Global Stocktake, a 1. It is a data centre facility where third partiescan
periodic review of global action, to achieve the lease space for serv ers and other computer
objectives of the Paris Agreement. hardware.

Which of the abov e statement(s) is/are correct? 2. They provide infrastructure like power supply,
bandwidth and cooling for setting up servers
a. 1 and 3 only and storage of data.

b. 1 and 2 only Which of the abov e statement(s) is/are incorrect?

c. 2 and 3 only a. 1 only

d. 1 , 2 and 3 b. 2 only

c. Both 1 and 2

3) Consider the following statements with respect to the d. Neither 1 nor 2


Kaladan Multi Modal Transit Transpor t Project:

1. It is joint initiativ e between India, My anmar


and Bangladesh. Answers
2. The project will connect the Kolkata Port with 1. c
My anmar’s Sittwe Port through the Dawki land
port in Bangladesh. Genetically Modified Vs Gene Editing
T echniques
Which of the abov e statement(s) is/are correct?
China approves safety of first gene -edited crop.
a. 1 only
 To create genetically modified crops and
b. 2 only animals, scientists will ty pically remove the
preferred gene from one organism and
c. Both 1 and 2 randomly introduce it into another organism.

d. Neither 1 nor 2  A well-known genetically modified ty pe of crop


is Bt corn and cotton, where a bacterial gene
was introduced that produces insecticidal
tox ins into the part of the plant where the insect
4) Consider the following statements with respect to eats, causing death to the insect.
Kremlin:
 In simple terms, gene editing is a small,
1. Kremlin means a fortress inside a city and controlled tweak to a liv ing organism’s ex isting
represents Russian political authority. DNA v ersus the introduction of a new, foreign
gene.
2. There are more than one Kremlins in Russia
and the one in Moscow is situated on the  It is nearly impossible to detect whether an
northern bank of the Moskv a Riv er. organism’s DNA has been edited or not because
the changes are indistinguishable from
naturally occurring mutations.

CHENNAI |SALEM| MADURAI | COIMBATORE DELHI | BANGALORE | T HIRUVANANTHAPURAM


www.shankariasacademy.com 16
www.iasparliament.com

 CRISPR is a common method, or tool, of gene enable us to reach the objectives set out in the
editing. Paris Agreement.

 The science behind CRISPR is detailed and  This is the first Global Stocktake y ear since the
complex, but it is a naturally occurring enzyme Paris Agreement was signed in 201 5 and the
in bacteria that allows scientists to edit DNA report has been underway for the past two
with precision. y ears.

 It is set to be released in September of 2023.

3. d

Kaladan Multi Modal Transit Transport Project

PM hails inaugural run of vessel from Syama Prasad


Mookerjee Port to Sittw e Port Myanmar developed
under Kaladan Multi Modal Transit Transport Project.

 It is a joint initiativ e of India and My anmar.

 It connects the Kolkata Port of India with the


Sittwe Port of My anmar by sea, Sittwe to
Paletwa v ia Riv er Kaladan, Paletwa to the
border of India, and My anmar v ia road and
2. a further ahead to Lawngtlai, Mizoram by road.
Petersberg Dialogue on Climate Change  The project was launched under the “Look East
Policy ” in 1 991.
 It has been hosted by Germany since 2010.
 Currently, the gov ernment undertook this as
 The dialogue is a forum for informal high-level
“Act East” remodelled policy.
political discussions, focusing both on
international climate negotiations and the  It is named a multimodal project as it uses a
adv ancement of climate action. wide range of infrastructures like roads,
bridges, and floating barrages.
 The Petersberg Dialogue on Climate Change
was held in Berlin from May 2-3, 2023.

 It was hosted by Germany and the United Arab


Emirates, which is hosting the 28th Conference
of Parties (COP28) to the United Nations
Framework Convention on Climate Change.

 Ministers from 40 countries attended the


conference to discuss the way forward towards
COP28.

 The UN Secretary reiterated the call for an


Acceleration Agenda, where all countries hit
fast-forward on their Net Zero deadlines.
4. d
 The Agenda calls for coal phase-out by 2030 in
Organisation for Economic Co -operation and Krem lin
Dev elopment countries, and by 2040 in all
 The word literally means “fortress inside a city”,
others.
and there are ov er 20 Kremlins across Russia.
 It also proposes for achieving Net Zero
 Howev er, the most important and popular one
electricity generation and decarbonising major
is in Moscow, metonymically used to refer to
sectors.
Russian political authority.
Global Stocktake
 It has been the seat of political power in Russia
 2023 is the y ear for the Global Stocktake, a since the 1 4th century (with a hiatus between
periodic review of global climate action which 1 7 13 and 1 918 when the czar shifted the capital
aims to assess whether current efforts will to St Petersburg).

CHENNAI |SALEM| MADURAI | COIMBATORE DELHI | BANGALORE | T HIRUVANANTHAPURAM


www.shankariasacademy.com 17
www.iasparliament.com

 It is situated on the northern bank of the 2. It supports the nationwide adoption of the
Moskv a Riv er. Integrated Coastal Zone Management (ICZM)
approach.
 It comprises fiv e palaces and four cathedrals,
and is enclosed within the Kremlin Wall with its 3. ICZM is a dy namic, multidisciplinary process to
distinctive towers. promote sustainable management of coastal
zones adopted in Rio de Janeiro, Earth summit.
 To the east of the Kremlin are the iconic St
Basil’s Cathedral and Red Square, where major Which of the statements giv en above is/are correct?
state functions take place.
a. 1 and 3 only
 The Kremlin has been a UNESCO World
Heritage Site since 1 990. b. 2 and 3 only

c. 2 only
5. c

Co-lending d. 1 , 2 and 3

 Co-lending is an arrangement where the loan


origination is by one entity but the risk is shared 2) Consider the following statements with respect to
by two entities. Arab League
 A non-banking financial company (NBFC) is 1. It is a regional organization of Arab states in the
the originator while a bank is where the major Middle East and parts of Africa.
portion of the loan rests.
2. It aims to strengthen and coordinate the
 One can call co -lending a blend of the co- political, cultural, economic, and social
origination and securitisation models. programs of its members and to mediate
disputes among them.
 The loan product could range from housing,
including affordable housing, gold, commercial 3. India is only granted the observer status to the
v ehicles, passenger v ehicles or even league.
microfinance loans (everything retail).
Which of the statement(s) giv en above is/are correct?
 An NBFC can partner with multiple banks and
a bank can partner with multiple NBFCs. a. 1 only

 There is no requirement for exclusivity. b. 2 and 3 only

 At this stage it may seem v ery similar to c. 1 and 2 only


securitisation contracts, but where co -lending
differs is that each bank and NBFC, based on d. 1 , 2 and 3
their internal practices and risk management
framework, formulates its own policy on co-
lending. 3) Which the following are the Tribes that are inhabited
in Kerala?
 The most binding aspect of this structure is the
risk-sharing code, 80% of the loan risk is borne 1. Cholanaikkans
by the bank and the rest by the NBFC.
2. Malay arayan
 Therefore, it enforces accountability on the
NBFC, unlike in a securitisation contract where 3. Erav allan
the NBFC’s role is more like a collection agent.
4. Kharia

5. Bhutia
08-05-2023
Select the correct answer using the codes
1 ) Consider the following statements with respect to
National Centre for Sustainable Coastal Management a. 1 , 2 and 3 only
(NCSCM)
b. 2 and 5 only
1. It is an autonomous institution established by
the Ministry of Fisheries, Animal Husbandry c. 1 , 2, 3 and 4 only
and Dairy ing.
d. 1 , 2, 3, 4 and 5

CHENNAI |SALEM| MADURAI | COIMBATORE DELHI | BANGALORE | T HIRUVANANTHAPURAM


www.shankariasacademy.com 18
www.iasparliament.com

4) Consider the following statements  It is approved by the Central government and


located in Chennai.
1. India’s Agricultural Trade Surplus is constantly
increasing from 2012-13 to 2022-23.  Aim - To support the protection, conservation,
rehabilitation, management, and policy advice
2. Import of Agricultural products have been of the coast.
gradually decreased for the past 5 y ears.
 Vision - To promote sustainable coasts
3. Crude oil imports in India is rapidly decreasing through increased partnerships, conservation
across the 5 y ears. practices, scientific research, and knowledge
management for current and future
Which of the statement(s) giv en above
generations.
is/are not correct?
 Approach - It supports the nationwide
a. 1 only
adoption of the Integrated Coastal Zone
b. 1 and 3 only Management (ICZM) approach.

c. 2 and 3 only  The Gov ernment of India embarked upon the


Integrated Coastal Zone Management Project
d. 1 , 2 and 3 (ICZMP) for the holistic dev elopment of the
coast.

 It is based on the regulatory framework of


5) Consider the following statements with respect to Coastal Regulation Zone (CRZ) Notification,
ty pes of Wildfires 201 1 and the Island Protection Zone (IPZ)
Notification, 2011.
1. Crown fires are the most intense and dangerous
wildland fires. Scientific Divisions of NCSCM

2. Ground fires burn only surface litter and duff  Coastal Env ironmental Impact Assessment
whereas Surface fires occur in deep (CIA)
accumulations of humus, peat and similar dead
v egetation.  Integrated Island Management (IIM)

3. Punjab had the highest number of wildfire  Coastal and Marine Resources Conserv ation
incidences in India. (CMR)

Which of the statements giv en above is/are correct?  Integrated Social Sciences and Economics (ISE)
a. 1 only  Futuristic Research (FTR)
b. 2 and 3 only
 Knowledge, Gov ernance and Policy (KGP)
c. 1 and 3 only
 Geospatial Sciences (GEO)
d. 1 , 2 and 3
2. c

Recently, Arab government representatives in Cairo


Answers (Egypt) voted to return Syria to the Arab League after
a 1 2-year suspension.
1. b
Arab League
Beachgoers in Chennai take part in green pledge and
signature campaign against littering and the need to  It is a regional organization of Arab states in the
live in harmony with nature Middle East and parts of Africa.

National Centre for Sustainable Coastal  It is formed in Cairo (Egy pt) on 1 945, as an
Managem ent (NCSCM) outgrowth of Pan-Arabism.

 It is an autonomous institution.  Aim - To strengthen and coordinate the


political, cultural, economic, and social
 Ministry – It is established by the Ministry of programs of its members and to mediate
Env ironment, Forest and Climate Change disputes among them or between them and
(MoEF&CC) in 201 1. third parties.

CHENNAI |SALEM| MADURAI | COIMBATORE DELHI | BANGALORE | T HIRUVANANTHAPURAM


www.shankariasacademy.com 19
www.iasparliament.com

 Founding Members - Egy pt, Sy ria, Lebanon, Kharia


Iraq, Jordan, Saudi Arabia, and Y emen.
 Kharia tribes hav e mainly concentrated in
 Currently, the League has 22 members. Jharkhand, they can also be found in Odisha
and West Bengal.
 Sy ria's participation was suspended from 2011
to 2023, it was reinstated recently.  Being a part of the 'Proto -Austroloid' family
group, the language of the Kharia tribes are
 The signing on April 1 3, 1 950, of an agreement known as Kharia language.
on joint defense and economic cooperation also
committed the signatories to coordination of  The major source of liv elihood for the Kharia
military defense measures. tribe is agriculture.

 In its early y ears the Arab League concentrated Bhutia


mainly on economic, cultural, and social
programs.  It is a community of Sikkimese people living in
the state of Sikkim.
 In 1 959 it held the first Arab petroleum
congress and in 1 964 established the Arab  They speak Sikkimese, a Tibetic language fairly
League Educational, Cultural and Scientific mutually intelligible with standard Tibetan.
Organization (ALECSO).
4. d
3. a
India’s farm trade surplus w as low er in 2022 -23 than
T ribes of Kerala in the previous year.

Cholanaikkans Food Price Index (FPI)

 The Cholanaikkans are an ethnic group of  It is released by UN Food and Agriculture


India. Organization.

 They primarily inhabit the southern Kerala  It is a measure of the monthly change in
State, especially Silent V alley National Park. international prices of a basket of food
commodities.
 They are one of the last remaining hunter-
gatherer tribes of the region.  It crashed from an av erage of 1 1 9.1 points in
201 3-14 to 90 points in 201 5-16.
Malay arayan
T rends in Imports and Ex ports (2022-23)
 They are members of a tribal community in
parts of Kerala state.  Department of Commerce data shows that
Total farm ex ports and imports during 2022-23
 Majority of the Population follows Hindu is higher than their previous year’s records.
religion.
 Agricultural Trade Surplus has marginally
 Some of the Malai Aray ans turned their dipped because of global prices and domestic
religious belief to Christianity. food inflation.

 They usually practice agriculture, but most of  The FPI has increased gradually across the
them lost their farm land due to ex ploitation. y ears.

Erav allan Major Ex port Items (2022-23)

 They are inhabitants of the Palghat District,  Marine products, Rice, Sugar, Basmati Rice,
Kerala. Spices, Buffalo meat, Raw cotton, Oil meals,
Wheat, Fruits and V egetables.
 They are either agriculturists or farm labourers.
Major Im port Items (2022-23)
 Ev en now there are many who practice
primitiv e animism, and worship animals, trees  V egetable oils, Fresh fruits, Pulses, Spices,
and stones and claim origin from them. Cashew, Raw Cotton, Natural rubber.

Crude oil im ports in 2022-23 rose y ear-on-year


gradually.

CHENNAI |SALEM| MADURAI | COIMBATORE DELHI | BANGALORE | T HIRUVANANTHAPURAM


www.shankariasacademy.com 20
www.iasparliament.com

5.c 09-05-2023
Recently, Wildfires occurred on inaccessible hilltops in 1 ) Consider the following statements with respect to
the Mhadei Wildlife Sanctuary. Saint Samarth Ramdas

Wildfire 1. He was born in Maharastra and a contemporary


of Sant Tukaram.
 It is also called as forest, bush or vegetation fire.
2. Dasbodh, Panchsamasi, Nirgundhy an and
 It can be described as any uncontrolled and v asudhaiva kudumbakam are some of his
non-prescribed combustion or burning of literary works.
plants in a natural setting such as a forest,
grassland, brush land or tundra. Which of the statement(s) giv en above is/are correct?

 Wildfire can be incited by human actions, such a. 1 only


as land clearing, ex treme drought or in rare
b. 2 only
cases by lightning (IRDR).
c. Both 1 and 2
 There are 3 conditions that need to be present
in order for a wildfire to burn: fuel, oxygen, and d. Neither 1 nor 2
a heat source.

Causing Factors
2) Consider the following statements
 Deficient Rainfall
1. Article 342(1 ) env isages that Supreme Court
 Unusual High Temperatures can designate a community as Scheduled Caste
or Scheduled Tribe.
 Low Moisture
2. Prov isions relating to the administration of
 Humidity tribal areas in Manipur is giv en in the 6th
Schedule of the Indian Constitution.
T y pes of Wildfires
3. President can declare any such area or the
 Crown fires – It burn trees up their entire whole state as a scheduled area.
length to the top.
Which of the statement(s) is /are correct?
 These are the most intense and dangerous
wildland fires. a. 1 only

 Surface fires – It burn only surface litter and b. 1 and 3 only


duff.
c. 3 only
 These are the easiest fires to put out and cause
the least damage to the forest. d. 1 , 2 and 3

 Ground fires – It occur in deep


accumulations of humus, peat and similar dead
3) Consider the following statements with respect to
v egetation that become dry enough to burn.
Gandak Riv er
 These fires mov e v ery slowly, but can become
1. It is formed by the union of the Kali and Trisuli
difficult to fully put out, or suppress. riv ers, which rise in the Great Himalay a Range
in Nepal.
 It is also called as underground or subsurface
fires. 2. It is the left bank tributary of Brahmaputra
Riv er.

3. It is the second breeding site for gharials in


India after the Chambal sanctuary.

Which of the statement(s) giv en above is/are correct?

a. 2 only

b. 1 and 2 only

CHENNAI |SALEM| MADURAI | COIMBATORE DELHI | BANGALORE | T HIRUVANANTHAPURAM


www.shankariasacademy.com 21
www.iasparliament.com

c. 1 and 3 only  He was a Hindu saint, philosopher, poet, writer


and spiritual master.
d. 1 , 2 and 3
 A dev otee of Lord Ram and Hanuman, he
toured the entire Indian subcontinent for 12
y ears.
4) Consider the following statements with respect to UN
Economic and Social Commission for Asia and the  Dasbodh, Panchsamasi, Nirgundhy an and
Pacific (ESCAP) v asudhaiva kudumbakam are some of his
literary works.
1. It is one of the 5 regional commissions of the
United Nations.  He was the one who established 1 100+ Mathas
across the Indian subcontinent.
2. It works under the jurisdiction of the United
Nations Economic and Social Council.
 He raised the importance of Sanatan Dharm
3. India is a member to the council. during his time.

Which of the statement(s) giv en above is/are correct?  The likes of Lokmany a Tilak, RSS founder KB
Hedgewar, and V D Sav arkar, all claimed to
a. 1 and 2 only hav e been inspired by the 17 th century saint.

b. 3 only 2. c

c. 1 and 3 only Chief Justice of India, D.Y . Chandrachud said that no


court or State has pow er to add, subtract or modify
d. 1 , 2 and 3 w ith the Scheduled Tribes List in Manipur

Scheduled Tribes Status with respect to


Manipur
5) Consider the following statements with respect to
Commission for Scientific and Technical Terminology  6th schedule - Prov isions relating to the
(CSTT) administration of tribal areas in the states of
Assam, Meghalay a, Tripura and Mizoram are
1. It has the mandate of ev olving technical giv en in this schedule but Manipur is not
terminology in all Indian languages. included in that.

2. It works under the aegis of the Union Ministry  Manipur’s Meitei Tribes are protested across
of Education. the state for inclusion of that community in the
Scheduled Tribe list.
Which of the statement(s) giv en abov e is/are
correct?Which of the statement(s) giv en abov e is/are  5th Schedule - Prov isions relating to the
correct? administration and control of scheduled areas
and scheduled tribes.
a. 1 only
 President can dec lare any such area or the
b. 2 only
whole state as a scheduled area.
c. Both 1 and 2
 Article 342(1) of the Constitution clearly says
d. Neither 1 nor 2 that President can designate a Scheduled Caste
or Scheduled Tribe.

 It is not open to State gov ernments or courts or


Answers tribunals or any other authority to modify,
amend or alter the list of Scheduled Tribes.
1. c
3. c
Saint Sam arth Ram das
The Gandak River became the second successful
 He is also known as Sant Ramdas or Ramdas breeding site for gharials in India after the Chambal
Swami. sanctuary

 He was born of Sury aji Panth and Renuka Bai Gandak Riv er
in Jamb, Maharashtra, in 1 608 A.D.
 Gandak Riv er, also called Narayani Riv er, river
 Ramdas was a contemporary of Sant Tukaram. in central Nepal and northern India.

CHENNAI |SALEM| MADURAI | COIMBATORE DELHI | BANGALORE | T HIRUVANANTHAPURAM


www.shankariasacademy.com 22
www.iasparliament.com

 It is formed by the union of the Kali and Trisuli 5.c


riv ers, which rise in the Great Himalay a Range
in Nepal; from this junction to the Indian The Commission for Scientific and Technical
border the river is called the Narayani. Terminology (CSTT) is to create technical and
scientific terminology in 1 0 Indian languages
 It flows southwest into India and then turns
southeast along the Uttar Pradesh–Bihar state Com mission for Scientific & T echnical
border and across the Indo -Gangetic Plain. T erminology (CSTT)

 It joins the Ganges northeast of Munger and  The Commission for Scientific and Technical
became the left bank tributary. Terminology (CSTT) works under the aegis of
the Union Ministry of Education.

 The CSTT was set up in 1 961 under clause (4) of


Article 344 of the Constitution of India as a
follow up of recommendations of a Committee
in this regard.

 Headquarters - New Delhi and has 22 State


Grant Academies / State Tex t-Book Boards /
Univ ersities Cells, etc.

 The CSTT has the mandate of evolving technical


terminology in all Indian languages.

4. d  Aim - To ev olve and define scientific and


technical terms in Hindi and all Indian
A new study says most countries in Asia and the Pacific languages; publish glossaries, definitional
are not prepared to manage extreme weather. dictionaries, encyclopedia.
United Nations Economic and Social
Com mission for Asia and the Pacific (ESCAP)
10-05-2023
 Aim - It aims to increase economic activity in
Asia and the Far East. 1 ) Consider the following statements with respect to
Enforcement Directorate (ED)
 It is the most inclusive intergovernmental
platform in the Asia-Pacific region. 1. It is assigned with inv estigation of offence of
money laundering and v iolations of foreign
 The Commission promotes cooperation among ex change laws.
its 53 member States and 9 associate members
in pursuit of solutions to sustainable 2. It works under the aegis of the Department of
dev elopment challenges. Rev enue, Ministry of Finance.

 ESCAP is one of the 5 regional commissions of 3. Director of ED is appointed under the Delhi
the United Nations. Special Police Establishment Act, 1 946.

 ESCAP also prov ides support to partners at the Which of the statement(s) giv en above is/are correct?
national lev el.
a. 1 and 2 only
 The commission is composed of 53 Member b. 1 only
States and nine Associate members, mostly
from the Asia and Pacific regions. c. 2 and 3 only
 In addition to countries in Asia and the Pacific, d. 1 , 2 and 3
the commission's members includes France,
the Netherlands, the United Kingdom and the
United States.
2) Consider the following statements with respect to
Statutory Liquidity Ratio (SLR) & Cash Reserv e Ratio
(CRR)

1. CRR is a minimum percentage of deposits that


a commercial bank has to maintain in the form
of liquid cash, gold o r other securities.

CHENNAI |SALEM| MADURAI | COIMBATORE DELHI | BANGALORE | T HIRUVANANTHAPURAM


www.shankariasacademy.com 23
www.iasparliament.com

2. SLR is a minimum percentage of deposits that 5) Consider the following statements with respect to
a commercial bank has to hold as reserves with Ex obiology Ex tant Life Surv eyor robot (EELS)
the central bank.
1. It is an autonomous robot that ex plores the
3. If CRR is high the inflation will increase and inaccessible locations on Earth and bey ond.
CRR is low then inflation will decrease.
2. It is designed jointly by the NASA and the
Which of the statement(s) giv en above European Space Agency (ESA).
is/are not correct?
3. It found the presence of water on Saturn’s moon
a. 1 only Enceladus.

b. 1 and 2 only Which of the statement(s) giv en above is/are correct?

c. 2 and 3 only a. 1 only

d. 1 , 2 and 3 b. 1 and 2 only

c. 2 and 3 only

3) Consider the following statements with respect to d. 1 , 2 and 3


Ex tended Producer Responsibility (EPR)

1. It specifies that a producer is responsible for a


product’s post-consumer stage. Answers
2. It is only applicable for non-recyclable 1. a
products.
The Supreme Court recently said it might revisit its
3. EPR is managed by the Plastic Waste 2021 ruling of tenure of a superannuated officer
(Management and Handling) Rules, 201 1.
Enforcement Directorate (ED)
Which of the statement(s) giv en above is/are correct?
 It is a law enforcement agency and economic
a. 2 only intelligence agency responsible for enforcing
economic laws and fighting economic crime in
b. 1 and 2 only India.

c. 1 and 3 only  It is part of the Department of Rev enue,


Ministry of Finance.
d. 1 , 2 and 3
 It is a multi-disciplinary organization

 The statutory functions of the Directorate


4) Consider the following statements with respect to include enforcement of following Acts:
Generativ e AI vs Conv ersational AI
1. The Prev ention of Money Laundering Act, 2002
1. Conv ersational AI engage in conversation and (PMLA)
refers to tools that allow users to communicate
with v irtual assistants or chat-bots. 2. The Foreign Ex change Management Act, 1999
(FEMA)
2. Generativ e AI uses generative adversarial
networks (GANs) to identify patterns and 3. The Fugitiv e Economic Offenders Act, 2018
features in a giv en dataset like ChatGPT. (FEOA)

3. India is party to the Global Partnership on 4. The Foreign Ex change Regulation Act, 197 3
Artificial Intelligence (GPAI). (FERA)

Which of the statement(s) giv en above is/are correct? 5. Sponsoring agency under COFEPOSA

a. 1 and 2 only T he concern of ex tending the term of


superannuated officer
b. 2 only
 Bey ond superannuation (outdated) -
c. 1 and 3 only There is an implied ex tension for an officer
appointed to one of these protected posts if the
d. 1 , 2 and 3

CHENNAI |SALEM| MADURAI | COIMBATORE DELHI | BANGALORE | T HIRUVANANTHAPURAM


www.shankariasacademy.com 24
www.iasparliament.com

appointment comes within 2 y ears of 3. c


retirement.
Recently, central government has introduced a draft
 The Centre promulgated an ordinances to notification on Extended Producer Responsibility
ex tend the tenure of the Enforcement (EPR) on w aste oil.
Directorate from 2 y ears to up to 5 y ears and
issued an order to amend the Fundamental Ex tended Producer Responsibility (EPR)
Rules, 1 922.
 It is an env ironmental policy approach in which
 But the SC said that ex tension of tenure should a producer’s responsibility for a product is
be done only in rare and exceptional cases and ex tended to the post-consumer stage of a
only for a short period. product’s life cy cle.

 It also made it clear that no further ex tension  It helps adv ance the circular economy,
shall be granted to him. decreases the env ironmental impact from a
product and its packaging.
2. d
 It promotes the principle of “polluter pays” by
The Reserve Bank of India’s (RBI) gold reserves holding the producer accountable for the entire
touched 7 94.64 metric tonnes in fiscal 2023. lifecy cle of the product.

Statutory Liquidity Ratio (SLR)  India first introduced EPR in 201 1 under
the Plastic Waste (Management and
 It is a minimum percentage of deposits that a Handling) Rules, 201 1 and E- Waste
commercial bank has to maintain in the form of Management and Handling Rules, 2011.
liquid cash, gold or other securities.
 EPR shifts the economic burden of the cost of
 It is basically the reserv e requirement that disposal from the gov ernment to the producer
banks are ex pected to keep before offering of the product.
credit to customers.
 The objectives of EPR are as follows
 These are not reserved with the Reserve Bank of
India (RBI), but with banks themselves. o Integration of env ironmental costs

Cash Reserv e Ratio (CRR) o Improved waste management

 Under cash reserve ratio (CRR), the o Reduction of disposal


commercial banks hav e to hold a certain
minimum amount of deposit as reserves with o Reduction of burden on municipalities
the central bank.
o Design of env ironmentally sound
 The percentage of cash required to be kept in products
reserves as against the bank's total deposits, is
called the Cash Reserv e Ratio. 4. d

Generative AI v s Conversational AI
 The cash reserve is either stored in the bank’s
v ault or is sent to the RBI. Conv ersational AI
 Banks can’t lend the CRR money to corporates  It is an AI that can engage in conversation and
or indiv idual borrowers, banks can’t use that refers to tools that allow users to communicate
money for inv estment purposes. with v irtual assistants or chatbots.
 And Banks don’t earn any interest on that  They mimic human interactions by identifying
money . speech and tex t inputs and translating their
contents into other languages.
 If CRR is high, bank deposits with RBI increase,
leading to a decrease in the bank’s capacity to  Com ponents - Natural language processing
lend. (NLP) and machine learning
 Hence, the interest rate increases as borrowing
 To keep the AI algorithms up-to-date, these
become ex pensive, and the market’s money NLP operations interact with machine learning
flow decreases inflation. processes in a continual feedback loop.

CHENNAI |SALEM| MADURAI | COIMBATORE DELHI | BANGALORE | T HIRUVANANTHAPURAM


www.shankariasacademy.com 25
www.iasparliament.com

 The fundamental elements of conversational AI 3. Stillbirth is the death or loss of a baby before or
enable it to process, comprehend, and produce during deliv ery.
responses naturally.
Which of the statement(s) giv en above is/are correct?
Generative AI
a. 2 and 3 only
 It often uses deep learning techniques, like
generative adv ersarial networks (GANs), to b. 1 and 2 only
identify patterns and features in a giv en dataset
before creating new data from the input data. c. 3 only

d. 1 , 2 and 3
 It consists of algorithms, features, and data sets
that constantly get better with use.

 The AI platform machine gets better at 2) Consider the following statements with respect to
identify ing patterns and employs them to create Uniform Civ il Code (UCC)
predictions as the input increases.
1. It is mentioned in the article 43 of the Indian
5. a Constitution.
NASA is testing a snake-like robot that could one day 2. It is enforceable by courts and justiciable in
explore the subsurface oceans on Saturn’s moon nature.
Enceladus
3. Goa is the only state which have uniform civil
T he Ex obiology Extant Life Surv eyor or EELS code in India.
 It is a robot is designed to autonomously map, Which of the statement(s) giv en above is/are correct?
trav erse and ex plore otherwise inaccessible
destinations on our planet, the Moon and other a. 1 only
planets in the solar sy stem.
b. 2 and 3 only
 EELS 1.0 - it weighs around 100 kilograms and
is 4 metres long. c. 3 only

 It is made of 1 0 identical segments that rotate d. 1 , 2 and 3


using screw threads for traction, grip and
propulsion.

 A radio signal to Saturn, for instance, could take 3) Consider the following pairs of leaders and their
around 83 minutes to travel in one direction. region in the 1 857 Revolt

 EELS will hav e to trav el while autonomously Leaders Region


sensing its env ironment and calculating risk to
1. Kunwar Singh - Kanpur
gather data with science instruments.
2. Nana Saheb - Bihar
 EELS creates a 3D map of its surroundings
using 8 stereo cameras and LIDAR. 3. Begum Hazrat Mahal - Awadh

 Nav igation algorithms use the data from those 4. Khan Bahadur Khan Rohilla - Uttar Pradesh
sensors to figure out the safest path for the
robot. Which of the abov e giv en pairs are correctly matched ?

a. 1 ,2 and 3 only

11-05-2023 b. 3 only
1 ) Consider the following statements with respect to c. 3 and 4 only
Neonatal Mortality Rate (NMR) & Maternal Mortality
Rate (MMR) d. 1 , 2, 3 and 4

1. NMR refers to the number of deaths under 1


y ear per 1 ,000 liv e births.
4) Consider the following statements
2. MMR is the number of deaths during a given
time period per 1 ,000 liv e births during the 1. Transformers in a computer sy stem translate a
same time period. sentence from one language to another.

CHENNAI |SALEM| MADURAI | COIMBATORE DELHI | BANGALORE | T HIRUVANANTHAPURAM


www.shankariasacademy.com 26
www.iasparliament.com

2. The decoder in the transformer ingests the  Neonate - Infants 0 to 27 days of life, under 28
input sentence and the encoder generates the day s of life.
translated sentence.
 Early neonate - Infants 0 to 6 day s (under 7
3. The capital T in ChatGPT corresponds to day s) of age.
transformers.
 Late neonate - Infants 7 to 27 days (under 28
Which of the abov e statement(s) is/are correct? day s) of age.
a. 1 and 2 only  Neonatal m ortality rate - [annual number
of death of infants 0 to 27 day s of life / the
b. 2 and 3 only annual number of liv e births] × 1 ,000.
c. 1 and 3 only
 It is declined in India from 201 3 but the rate of
d. 1 , 2 and 3 decline has been slow and lags behind that of
infant and under-fiv e child mortality rates.

Maternal Mortality Rate


5) An assessment on climate change ov er the Indian
subcontinent by the Ministry of Earth Science, found  MMR is the number of maternal deaths during
that India is heating up slower than the world average. a giv en time period per 1 00,000 liv e births
In this context, identify the reasons for India’s relative during the same time period.
slow heating.
 Maternal mortality in a region is a measure of
1. Land areas hav e a tendency to get heated faster, the reproductive health of women in the area.
and by a larger amount, than oceans.
 Many women in reproductive age-span die due
2. Oceans hav e a higher capacity to cool to complications during and following
themselves down through the process of pregnancy and childbirth or abortion.
ev aporation.
 A maternal death occurred almost ev ery two
3. The increase in the temperature is high in minutes in 2020.
higher altitudes, near the Polar Regions, than
near the equator.  Between 2000 and 2020, the maternal
mortality ratio (MMR, number of maternal
Which of the statement(s) giv en above is /are correct? deaths per 1 00 000 liv e births) dropped by
about 34% worldwide.
a. 1 only
 Almost 95% of all maternal deaths occurred in
b. 2 and 3 only low and lower middle-income countries in
2020.
c. 3 only
2. c
d. 1 , 2 and 3
Uniform Civil Code (UCC)

 UCC is one that would provide for one law for


Answers the entire country, applicable to all religious
1. c communities in their personal matters such as
marriage, divorce, inheritance, adoption etc.
UN report says that India tops the list of 1 0 countries,
bear 60% of global maternal deaths, stillbirths &  It is mentioned in Article 44, a directive
new born deaths burden. principle of the Constitution.

Neonatal Mortality Rate (NMR)  It say s that the state shall endeav or to secure a
UCC for the citizens throughout the territory of
 NMR refers to the number of deaths of infants India.
aged 0–27 days per 1,000 live births.
 The directive principle are not justiciable (not
 Infant m ortality rates refer to the number of enforceable by any court) but the principles laid
deaths under age 1 y ear per 1 ,000 live births. down therein are fundamental in gov ernance.

 The NMR and IMR are commonly accepted as  Article 43 mentions state shall endeavor by
a measure of the general health and wellbeing suitable legislation while in Article 44, this
of a population. phrase is absent.

CHENNAI |SALEM| MADURAI | COIMBATORE DELHI | BANGALORE | T HIRUVANANTHAPURAM


www.shankariasacademy.com 27
www.iasparliament.com

 This implies that the duty of the state is greater algorithms to generate tex t responses to
in other directive principles than in Article 44. prompts.

 When enacted the code will work to simplify Machine learning (ML)
laws that are segregated at present on the basis
of religious beliefs like the Hindu code bill,  It is a subfield of artificial intelligence, teaches
Shariat law, and others. computers to solv e tasks based on structured
data, language, audio, or images, by providing
 The code will simplify the complex laws around ex amples of inputs and the desired outputs.
marriage ceremonies, inheritance, succession,
adoptions making them one for all. T ransformers in ChatGPT

 The same civ il law will then be applicable to all  A transformer is a two -part neural network.
citizens irrespective of their faith.
 The first part is an ‘encoder’ that ingests the
3. c input sentence in the source language (e.g.
English);
Sepoy Mutiny, 1857
 The second is a ‘decoder’ that generates the
 Indian Mutiny , also called Sepoy Mutiny or translated sentence in the target language
First War of Independence. (Hindi).

 Begun in Meerut by Indian troops (sepoys) in  The encoder converts each word in the source
the serv ice of the British East India Company, it sentence to an abstract numerical form that
spread to Delhi, Agra, Kanpur, and Lucknow. captures the meaning of the word within the
context of the sentence, and stores it in a
Causes memory bank.

 The doctrine of lapse by Lord Dalhousie in the  Both these processes use a mechanism called
late 1 840s. ‘attention’.

 British gov ernment’s decision to replace the  A key improvement ov er prev ious methods is
old-fashioned musket with the Enfield rifle. the ability of a transformer to translate long
sentences or paragraphs correctly.
 Locals believed the cartridges had a mix of fat
of cows and pigs with flour.  Transformers hav e also become popular in
computer v ision, they simply cut an image into
 Another serious concern was the increasing small square patches and line them up, just like
pace of Westernization, by which Hindu society words in a sentence.
was being affected by the introduction of
Western ideas.  Today , transformer models constitute the best
approach for image classification, object
 Missionaries were challenging the religious detection and segmentation, action
beliefs of the Hindus and muslims. recognition, and a host of other tasks.

Leaders 5. d

 Kunwar Singh –Bihar A recent report says that India’s Annual mean
temperatures slower than the w orld average
 Nana Saheb - Kanpur
India’s T emperature Criteria
 Begum Hazrat Mahal - Awadh
 Annual mean temperatures in India is slower
 Khan Bahadur Khan Rohilla - Uttar Pradesh when compared to world level

4. d  Most of India’s location is in the lower latitudes.

ChatGPT  A majority of the global landmass is


concentrated in the no rthern latitudes. In the
 It is a large language model dev eloped by tropics and along the equator, it is mostly
OpenAI that can be used for natural language oceans.
processing tasks such as tex t generation and
language translation.  Reasons - Land areas hav e a tendency to get
heated faster, and by a larger amount, than
 It is based on GPT-3.5 (Generativ e Pretrained oceans.
Transformer 3.5) model and uses deep learning

CHENNAI |SALEM| MADURAI | COIMBATORE DELHI | BANGALORE | T HIRUVANANTHAPURAM


www.shankariasacademy.com 28
www.iasparliament.com

 Oceans hav e a higher capacity to cool 2) Consider the following statements with respect to
themselves down through the process of disqualification of a Member of Parliament
ev aporation.
1. The speaker can disqualify a member of
 The warmer water evaporates, leaving the rest parliament under the 1 0th Schedule of Indian
of the ocean relatively cooler. Constitution.

 The increase in temperatures is known to be 2. The question of disqualification under the 10th
more prominent in the higher altitudes, near Schedule is decided by the Chairman in the case
the Polar Regions, than near the equator. of Rajy a Sabha and Speaker in the case of Lok
Sabha.
 This is because of an atmospheric phenomena,
including heat transfers from the tropics to the 3. The decision of the Chairman/Speaker in cases
poles through prev ailing sy stems of air of disqualify ing a member of parliament is
circulation. subjected to judicial review.

 India happens to be in the tropical region, quite Which of the statement(s) giv en above is/are correct?
close to the equator.
a. 1 and 2 only
 Aerosols – It refer to all kinds of particles b. 1 only
suspended in the atmosphere.
c. 2 and 3 only
 These particles have the potential to affect the
local temperature by absorbing heat and it d. 1 , 2 and 3
affect the cloud by sunlight’s reflection or
absorption.

 Aerosol concentration over the Indian region is 3) Consider the following statements with respect to
quite high, due to natural as well as man-made Monkey Pox (mpox)
reasons.
1. It is a zoonosis disease that transmitted from
infected animals to humans.

12-05-2023 2. It is an orthopoxvirus, a genus of v iruses that


includes v ariola v irus which causes smallpox.
1 ) Consider the following statements with respect to
Gov ernment of Delhi 3. WHO has recently ended the mpox as public
health emergency of international concern
1. It has the legislativ e and ex ecutive control over (PHEIC)
administrative services ex cept with regard to
public order, police and land. Which of the statement(s) giv en above are correct?

2. The ex ecutive power of the Union Territory of a. 1 and 2 only


Delhi is co-extensive with its legislative power.
b. 2 and 3 only
3. The Lieutenant Gov ernor shall be bound by the
decisions of the Gov ernment of Delhi on c. 1 and 3 only
serv ices.
d. 1 , 2 and 3
Which of the statement(s) giv en above is/are correct?

a. 1 and 2 only
4) Consider following statements with respect to Comb
b. 2 only Jellies

c. 1 and 3 only 1. Comb jellies, or ctenophores are one of the


oldest animal lineages with a defined nervous
d. 1 , 2 and 3 sy stem.

2. Neuron that form sy napses were all


interconnected by a single plasma membrane is
a unique feature in this species.

3. All ctenophores hav e two long tentacles but


they lack stinging cells.

CHENNAI |SALEM| MADURAI | COIMBATORE DELHI | BANGALORE | T HIRUVANANTHAPURAM


www.shankariasacademy.com 29
www.iasparliament.com

Which of the statement(s) giv en above is/are correct?  The Legislative Assembly shall hav e power to
make laws for the whole or any part of the
a. 1 only National Capital Territory with respect to any of
the matters enumerated in the State List or in
b. 2 and 3 only the Concurrent List.
c. 1 and 2 only  There shall be a Council of Ministers consisting
of not more than 1 0% of the total number of
d. 1 , 2 and 3
members in the Legislative Assembly, with the
Chief Minister at the head to aid and adv ise the
Lieutenant Gov ernor.
5) Consider the following statements with re spect to
Poly gamy 2. d

1. Section 494 of Indian Penal Code (IPC) Power of a Speaker in Disqualification of


penalizes bigamy or polygamy in India. Mem ber of Parliam ent (M.P)

2. The highest prev alence of poly gynous  Article 1 02 of Indian Constitution defines about
marriages was in the Northeastern states with the Disqualification of Members in Parliament.
tribal populations.
 A person shall be disqualified from
3. The prev alence of poly gamy was high among membership in either house of Parliament if he
Christians, than the Muslims and Hindus. is so disqualified under the Tenth Schedule of
the Indian Constitution.
Which of the statement(s) giv en above is/are correct?
 The Tenth Schedule of the Indian Constitution
a. 1 only speaks about Anti-defection law s.

b. 2 and 3 only  Disqualification on the ground of anti-defection


laws:
c. 1 and 2 only
 The person will be disqualified if they do any of
d. 1 , 2 and 3 the following:

o Giv ing up one's position in their party


v oluntarily.
Answers
o Acting in opposition to their party's
1. d
policies.
Legislative Powers of Union T erritory (DELHI)
o After being elected and joining another
 Unlike other Union Territories Union Territory political party.
of Delhi hav e some special provisions.
o If the person is a nominated member
and joins a political party after the six-
 Special Prov isions of the Union Territory of
month period has ex pired.
Delhi is enumerated in Part-v iii Under article
239AA. o If less than two-thirds of a political
party 's members join another party.
69th Am endment Act, 1991
 Speaker is the sole and final arbiter of deciding
 The Union territory of Delhi shall be called the
the disqualification of an M.P.
National Capital Territory of Delhi
 After Kihoto Hollohan v s Zachilhu case (1993),
 The administrator thereof appointed under
the Supreme Court declared that the decision of
article 239 shall be designated as the
the presiding officer is not final and can be
Lieutenant Gov ernor.
questioned in any court and it is subject to
judicial review.
 There shall be a Legislativ e Assembly for the
National Capital Territory
 Speaker is the deciding party for the
disqualification of an M.P after the resolution is
 The seats in such Assembly shall be filled by
passed for the giv en abov e reasons but
members chosen by direct election from
subjected to Judicial Rev iew.
territorial c onstituencies in the National
Capital Territory.

CHENNAI |SALEM| MADURAI | COIMBATORE DELHI | BANGALORE | T HIRUVANANTHAPURAM


www.shankariasacademy.com 30
www.iasparliament.com

3. d  Ctenophores hav e 8 comb rows of fused cilia


arranged along the sides of the animal, clearly
World Health Organization (WHO) said that it was v isible along the red lines.
ending a near year-long global health emergency for
mpox (MonkeyPox)  These cilia beat sy nchronously and propel
ctenophores through the water. Some species
 Monkey pox is a zoonosis, that is, a disease that mov e with a flapping motion of their lobes or
is transmitted from infected animals to undulations of the body.
humans.
 Many ctenophores hav e 2 long tentacles, but
 Monkey pox v irus infection has been detected in some lack tentacles completely.
squirrels, Gambian poached rats, dormice, and
some species of monkeys.  Ctenophores hav e a pair of anal pores, which
hav e sometimes been interpreted as
 Cause - Monkey pox virus. homologous with the anus of bilaterian animals
(worms, humans, snails, fish, etc.).
 While the natural reservoir of monkey pox
remains unknown, African rodents and  Habitat - Some ctenophores liv e in somewhat
monkey s are suspected of transmission and brackish water, but all are confined to marine
infection. habitats.

 Occurrence - According to the WHO, cases  They liv e in almost all ocean regions,
occur close to tropical rainforests inhabited by particularly in surface waters near shores.
animals that carry the v irus.
 At least two species (Pleurobrachia pileus and
 According to the WHO, two distinct clades are Beroe cucumis) are cosmopolitan, but most
identified hav e a more restricted distribution.
o The West African clade  Ex cept for one parasitic species, all of them are
carniv orous, eating myriads of small planktonic
o The Congo Basin clade (the Central animals.
African clade)
Recent Findings
 T ransmission - Transmission can occur
through contact with bodily fluids, lesions on  Neurites – the branches from the neuron that
the skin or on internal mucosal surfaces, such form sy napses – were all interconnected by a
as in the mouth or throat, respiratory droplets single plasma membrane, a feature not seen in
and contaminated objects. the neurons of other animals.
 Human-to-human transmission is limited and 5. d
the longest documented chain of transmission
is six generations. National Family Health Survey -5 (201 9-20) showed
the prevalence of polygamy in India
 The monkey pox v irus is an orthopoxvirus,
which is a genus of v iruses that also includes the Poly gamy
v ariola virus, which causes smallpox, and
v accinia virus, which was used in the smallpox  Poly gamy is the practice of hav ing more than
v accine. one married spouse — wife or husband.

 Monkey pox causes sy mptoms similar to  The issue is gov erned both by personal laws and
smallpox, although they are less severe. the Indian Penal Code (IPC).

4. c  T he Hindu Marriage Act, 1955 outlawed


the practice of polygamy.
A recent study published in Science found something
unexpected in the nerve system of the combjellies or  Buddhists, Jains, and Sikhs are also included
ctenophores. under the Hindu Marriage Code.

Com b Jellies  Parsi Marriage and Div orce Act, 1936 –It
outlawed bigamy.
 Comb jellies, or ctenophores, belong to phylum
ctenophora, and are one of the oldest animal  Shariat Act, 1937 - Marriage in Islam is
lineages with a defined nerv ous system. gov erned by the Shariat Act, 1 937.

CHENNAI |SALEM| MADURAI | COIMBATORE DELHI | BANGALORE | T HIRUVANANTHAPURAM


www.shankariasacademy.com 31
www.iasparliament.com

 Personal law allows a Muslim man to hav e four d. 1 , 2 and 3


wiv es.

 Any mov e to outlaw poly gamy for Muslims


would hav e to be a special legislation which 2) Consider the following statements with respect to
ov errides personal law protections like in the Carbon Dating
case of triple talaq.
1. Carbon dating is a widely used method to
IPC Section 494 determine the age of organic materials.

 Marry ing again during lifetime of husband or 2. Half - Life in carbon dating means the
wife penalises bigamy or polygamy. radioactive C-1 4 in organic materials reduces to
one half of itself while c -1 2 remains the same.
 This prov ision does not apply to a marriage
3. Potassium-argon dating and Uranium-
which has been declared v oid by a court for
thorium-lead dating are also the methods have
ex ample, a child marriage that has been
declared v oid. the same principle with respect to carbon
dating.
 IPC doesn’t apply to a spouse who has been
Which of the statement(s) giv en above is/are correct?
deserted the marriage and continually absent
for 7 y ears it will not bind the other spouse from a. 1 only
remarrying.
b. 1 and 2 only
National Fam ily Health Survey-5 (2019-20)
c. 1 and 3 only
 The National Family Health Surv ey-5 (2019-
20) showed the prev alence of polygamy d. 1 , 2 and 3
was 2.1% am ong Christians, 1 .9% among
Muslims, 1 .3% among Hindus, and 1 .6% among
other religious groups.
3) Consider the following statements with respect to
 The highest prev alence of poly gynous WHO Report on Pre-term Births
marriages was in the Northeastern states
with tribal populations. 1. Pre-term babies refers to babies born alive
before 37 weeks of pregnancy hav e been
 A list of 40 districts with the highest polygyny completed.
rates was dominated by those with high tribal
populations. 2. Preterm birth is the single largest killer of
children under 5 y ears of age according to the
WHO report, 2020.

13-05-2023 3. Maternal mortality and Neonatal and child


mortality comes under the Sustainable
1 ) Consider the following statements with respect to Dev elopment Goal 3.
Electronic V oting Machines (EV Ms)
Which of the statement(s) giv en above is/are correct?
1. An Electronic V oting Machine (EV M) is an
electronic device used for recording votes. a. 1 and 3 only

2. It is dev eloped by Election Commission of India b. 2 only


in collaboration with Bharat Electronics
Limited and Electronics Corporation of India c. 2 and 3 only
Limited.
d. 1 , 2 and 3
3. Although India does not import EV Ms from
abroad, foreign technologies are used in Indian
EV Ms.
4) Consider the following statements with respect to
Which of the statement(s) giv en above is/are correct? Moroccan Locust

a. 1 only 1. It was traditionally considered as one of the


most dangerous agricultural pests in the
b. 1 and 2 only Mediterranean zone.

c. 1 and 3 only

CHENNAI |SALEM| MADURAI | COIMBATORE DELHI | BANGALORE | T HIRUVANANTHAPURAM


www.shankariasacademy.com 32
www.iasparliament.com

2. The amount of spring rainfall is critical for the  EVM Parts - Control unit and a balloting unit
dev elopment cycle of this agriculture connected by a 5 -metre cable.
threatening species.
 An EV M comprises of a control unit which is
3. It has the capability to migrate in swarms from placed with the polling officer and a balloting
the Canary Islands to Afghanistan. unit which is placed inside the v oting
compartment.
Which of the statement(s) giv en above is/are correct?
 The control unit has been termed the EV M’s
a. 1 and 2 only ‘brain’.
b. 1 and 3 only  Voting - The balloting unit is turned on only
after the polling officer presses the Ballot
c. 3 only
button on it and the v ote is then cast.
d. 1 , 2 and 3
 The Software Programme Code is not
outsourced, and subjected to security
procedures at factory lev el to maintain the
5) Consider the following statements with respect to highest lev els of integrity.
United Nations Forum on Forests (UNFF1 8)
 The programme is converted into machine code
1. It is a subsidiary body established under and only then giv en to the chip manufacturer
Economic and Social Council of the United abroad.
Nations (ECOSOC) based on the Rio
Declaration.  Because we don’t hav e the capability of
producing semi-conductor microchips within
2. It is mandatory for UN members to adopt the country.
Strategic Plan of Global Forest Goals and
associated targets to be achieved by 2030.  Each ECI-EV M has a serial number and the
Election Commission by use of EV M-tracking
Which of the statement(s) giv en above is/are correct? software can find out from its database which
machine is located where.
a. 1 only
 Any attempt to replace a microchip is
b. 2 only detectable and can make EV M inoperative.
c. Both 1 and 2  The process of destruction of EV M & its chip is
carried out in the presence of the Chief
d. Neither 1 nor 2
Electoral Officer of the state or his
representatives inside the factory of
manufacturers.
Answers 2. d
1. b
Allahabad High Court ordered a scientific survey,
Election Commission of India (ECI) denied claims of including carbon dating, of a Shivling that have been
Randeep Surjewala, who had said that Electronic found at the Gyanvapi mosque complex, V ara nasi.
V oting Machines (EV Ms) from South Africa were being
used for voting. Carbon Dating

Electronic Voting Machines (EVMs)  Carbon dating is a widely used method to


determine the age of organic materials, things
 India does not use any EV Ms produced abroad. that were once living.

 Producers - EV Ms are produced indigenously  The dating method is based on Carbon-1 4 (C-
by 2 PSUs v iz. Bharat Electronics Ltd. and 1 4), an isotope of carbon with an atomic mass
Electronics Corporation o f India Ltd. of 1 4, is radioactive, and decays at a well known
rate.
 For the machines, the Software Programme
Code is written in-house, by these two  The most abundant isotope of carbon in the
companies, not outsourced, and subjected to atmosphere is C-1 2 and a v ery small amount of
security procedures at factory level to maintain C-1 4 is also present.
the highest lev els of integrity.
 The ratio of C-1 2 to C-1 4 in the atmosphere is
almost static, and is known.

CHENNAI |SALEM| MADURAI | COIMBATORE DELHI | BANGALORE | T HIRUVANANTHAPURAM


www.shankariasacademy.com 33
www.iasparliament.com

 Half Life - While C-1 2 is stable, the radioactive  In 2020, an estimated 1 3.4 million babies were
C-1 4 reduces to one half of itself in about 5,7 30 born preterm.
y ears known as its half-life.
 Preterm birth is the single largest killer of
 There are other methods to calculate the age of children under 5 years of age.
inanimate things that are more than 40,000-
50,000 years old because after 8-1 0 cy cles of  According to estimates, around 1 million
half-liv es, the amount of C-1 4 becomes almost newborns died due to preterm birth-related
v ery small. complications in 2020.

 Radiom etric Dating Methods - It is based  Preterm birth rates were 9.9% in 2020,
on the same principle as carbon dating and is compared to 9.8% in 201 0.
known as radiometric dating methods.
 In high-burden regions such as South Asia, it is
 Two commonly employed methods for dating increased in 2020 when compared to 2010.
rocks are potassium-argon dating and
uranium-thorium-lead dating.  In Sub-Saharan Africa it maintained as per
201 0 in 2020.
 The radioactive isotope of potassium decays
into argon, and their ratios can give a clue about  Pre-term birth rates reduced in Czech Republic,
the age of rocks. Austria, Brunei, Singapore, Spain, the
Netherlands, Denmark, Hungary , Brazil and
 Uranium and thorium have several radioactive Sweden.
isotopes, and all of them decay into the stable
lead atom.  Preterm birth is associated with long-term
damage to respiratory and cardiac sy stems for
 The ratios of these elements present in the many surv ivors and can also hav e a
material can be measured and used to make neurodevelopmental impact.
estimates about age.
 Disabilities can range from less severe
3. d outcomes to major disabilities like diplegia.

WHO report says that India recorded maximum 4. d


preterm births in 2020.
Food and Agriculture Organization (FAO) of the
WHO Report on Pre-term Births United States alerted that the large-scale outbreak of
the Moroccan locust can cause threat to across 8
 Babies born aliv e before 37 weeks of pregnancy provinces in Afghanistan.
are complete are called preterm babies.
Moroccan Locust
 Based on gestational age, there are 3 categories
of preterm births, as defined by the WHO:  The Moroccan locust, Dociostaurus
maroccanus was traditionally considered as one
o Ex tremely preterm (less than 28 of the most dangerous agricultural pests in the
weeks) Mediterranean zone.

o V ery preterm (28 to less than 32 weeks)  It has the capability to migrate in swarms made
it a major enemy of agriculturists from the
o Moderate to late preterm (32 to 37 Canary Islands to Afghanistan.
weeks)
 Climatic factors, in particular the amount of
 Preterm births can happen both spontaneously spring rainfall, are critical for the
or for sev eral medical reasons, including dev elopmental cycle of D. maroccanus.
infection, or other pregnancy complications
that require early induction of labour or  Anthropogenic factors appear to hav e the most
caesarean birth. powerful effect on the locust's population
dy namics.
Findings of the Report
 Deforestation and ov ergrazing create the
 According to the WHO report, rates of preterm necessary prerequisites for colonization by the
birth hav e barely changed between 201 0 and Moroccan locust.
2020.
 On the other hand, converting grasslands into
 In fact, it is ev en rising in some parts of the croplands makes the habitat totally unsuitable
world.

CHENNAI |SALEM| MADURAI | COIMBATORE DELHI | BANGALORE | T HIRUVANANTHAPURAM


www.shankariasacademy.com 34
www.iasparliament.com

for the insect because females can lay eggs into 15-05-2023
undisturbed soil.
1 ) Kudivaram, Pathuvaram and Aayathinel these
 These two conflicting tendencies appear to terms are sometimes mentioned in news belongs to 11th
gov ern the current ev olution of Moroccan century C.E., is related to which of the following
locust populations, their manifestations being options?
different according to the geographical zone.
a. Epic Literatures
 Although in some regions (e.g. in many
European countries) D. maroccanus has lost its b. Port Towns
formerly high economic importance.
c. Tax es
 North African and central Asian countries the d. Dams and Reserv oirs
species continues to flourish and may even
increase its pest status.

5. a 2) Consider the following statements with respect to


Palghat Gap
UNFF1 8 held in New Y ork, discussed the contributions
of SFM to energy, livelihoods and the SDGs. 1. It is a low mountain pass in the Western Ghats
between Coimbatore in Tamil Nadu and
United Nations Forum on Forests (UNFF18) Palakkad in Kerala.
 The UN General Assembly adopted the first 2. The Gap is a geological shear zone that runs
ev er UN Strategic Plan for Forests 2017-2030. from east to west.
 It is a subsidiary body established under 3. The Mullay ar riv er flows through it and the
Economic and Social Council of the United v egetation here is classified as dry ev ergreen
Nations (ECOSOC). forest.

 The Strategic Plan provides a global framework Which of the statement(s) giv en above is/are correct?
for actions at all levels to sustainably manage all
ty pes of forests and trees outside forests and a. 1 only
halt deforestation and forest degradation.
b. 1 and 2 only
 At the heart of the Strategic Plan are 6 Global
Forest Goals and 26 associated targets to be c. 2 and 3 only
achiev ed by 2030, which are v oluntary and
univ ersal. d. 1 , 2 and 3

 They support the objectives of the International


Arrangement on Forests.
3) Consider the following statements with respect to
Director of Central Bureau of Inv estigation (CBI)
 It aims to contribute to progress on the
1. Both the Prime Minister and the leader of
o Sustainable Dev elopment Goals,
opposition in Lok Sabha are in the committee of
o The Aichi Biodiv ersity Targets, appointing the Director of CBI.

o The Paris Agreement adopted under the UN 2. CBI deriv es its power to inv estigate from the
Framework Convention on Climate Change. Delhi Special Police Establishment Act, 1 946.

Quadrennial programme for the period 2017 - 3. CBI can conduct investigation in a state only
2020 with the consent of the concerned state
gov ernment.
 Odd-y ear sessions will focus on discussions on
Which of the statement(s) giv en above is/are correct?
implementation, technical advice and exchange
of ex periences.
a. 1 only
 Ev en-y ear sessions will focus on policy b. 2 and 3 only
dialogue, development and decision-making.
c. 1 and 3 only
 The Forum has univ ersal membership, and is
composed of all Member States of the United d. 1 , 2 and 3
Nations and specialized agencies.

CHENNAI |SALEM| MADURAI | COIMBATORE DELHI | BANGALORE | T HIRUVANANTHAPURAM


www.shankariasacademy.com 35
www.iasparliament.com

4) Consider the following statements with respect to  One of the inscription is engrav ed on a stone,
Dedicated Freight Corridors (DFC) written in the Tamil language and characters of
1 1 th century C.E.
1. It is a high-speed and high-capacity railway
corridor that is ex clusively meant for the  It is a record of a gift of land for a koneri (a
transportation of goods and commodities. square pond with steps from all the sides).

2. There are no dedicated freight corridors in  It further mentions some tax es such
South India. as kudivaram, pathuvaram and aayathinel
(paddy ).
3. It works under the aegis of Ministry of
Railway s.  The other is a Sundara Pandy a inscription that
is written in the Tamil language and characters
Which of the statement(s) giv en above is/are correct? of the 1 3th century C.E.
a. 1 only
 This is also an incomplete inscription and
b. 2 and 3 only recorded the sale of land owned by Akkanayan
and Thikkanai, sons of Tiruv enkadudaiyar and
c. 1 and 3 only Av vaiyan, residents of Tondaiman Arrur, for
1 00 panam.
d. 1 , 2 and 3
 It further states that the land was lo cated at two
different places— Tondailasv amikoyil and
Tirukkalathiy arsomapandin in Tondamanadu
5) Consider the following statements with respect to of Ariurnadu in Thiruv enkadu-Kottam.
Central Equipment Identity Register (CEIR) Sy stem
2. b
1. It is to ease reporting of stolen and lost mobiles
and block the use of mobiles all ov er the Palghat Gap
country.
 It is a low mountain pass in the Western Ghats
2. It is an initiativ e of technology development between Coimbatore in Tamil Nadu and
body , the Centre for Department of Telematics Palakkad in Kerala.
(CDoT).
 The Bharathappuzha river flows through it.
3. It can restrict the use of stolen mobiles by
second hand consumers.  In contrast to the tropical rainforests of the
Western Ghats, the v egetation in the Palghat
Which of the statement(s) giv en above is/are correct? Gap is classified as dry evergreen forest.

a. 1 only  Sev eral species of frogs are found only on one


side of the Gap.
b. 1 and 3 only
 Elephant populations on the Nilgiris side differ
c. 2 and 3 only
in their mitochondrial DNA from elephants in
the Anamalai and the Periy ar sanctuaries.
d. 1 , 2 and 3
 The Western Ghats in north of the Palghat Gap
receive more rain annually, but the south gets
rain more ev enly throughout the y ear.
Answers
1. c Geological Upheaval

2 inscriptions, written in Tamil from the 1 1 th century  The Gap is a geological shear zone that runs
and the 1 3th century C.E., were found in Tirupati from east to west.
district.
 Shear zones are weak regions in the earth’s
T am il inscriptions in T irupati crust this is the reason why tremors are
sometimes felt in the region Coimbatore.
 It is found from the ruins of an old Lord Shiva
temple at Tondamanadu v illage in Tirupati
district.

CHENNAI |SALEM| MADURAI | COIMBATORE DELHI | BANGALORE | T HIRUVANANTHAPURAM


www.shankariasacademy.com 36
www.iasparliament.com

of the single largest opposition party and Chief


Justice of India/ a Supreme Court Judge.

 CBI is ex empted from the prov isions of the


Right to Information Act.

 CBI is India's officially designated single point


of contact for connection with the Interpol.

4. c

Recently, Pipavav Port is connected to the Western


DFC (WDFC) through a double-stack rail netw ork

A dedicated Freight Corridor (DFC)

3. d

PM-led panel picks Karnataka DGP Praveen Sood as


next CBI Director.

Central Bureau of Investigation (CBI)

 The CBI was established on the


recommendation of Santhanam Committee on
Prev ention of Corruption.

 CBI is not a statutory body.

 Powers - It deriv es its power to inv estigate


from the Delhi Special Police Establishment
Act, 1 946.

 Com position - CBI is headed by a Director  It is a high-speed and high-capacity railway


and assisted by a special director or an corridor that is ex clusively meant for the
additional director. transportation of goods and commodities.

 Functions - CBI is the main inv estigating  With seamless integration of world-class
agency of the Central Gov ernment. technology, DFC ensures faster transit, reduced
logistics costs, higher energy efficiency and
Director of Central Bureau of Inv estigation env ironment-friendly operations.
(CBI)
 It is built to affirm a higher throughput per train
 The CBI Director is an IPS officer with the rank and a more significant share in the freight
of Director General of Police. market.
 It operates under the jurisdiction of the  Dedicated Freight Corridor (DFC) project was
Ministry of Personnel, Public Griev ances and conceived in 2005.
Pensions.
 6 DFCs
 The CBI Director was appointed on the basis of
the DSPE Act, 1 946 till 2014. o Eastern DFC (EDFC) – Sanctioned.

 After 201 4, Lokpal Act prov ided a committee o Western DFC (WDFC) – Sanctioned.
for the appointment of the CBI Director.
o East-Coast Corridor – Approved for
 The appointment committee consists of the preparing DPRs
Prime Minister, Leader of Opposition/ Leader

CHENNAI |SALEM| MADURAI | COIMBATORE DELHI | BANGALORE | T HIRUVANANTHAPURAM


www.shankariasacademy.com 37
www.iasparliament.com

o East-West Corridor - Approved for  It aids the gov ernment in prev enting revenue
preparing DPRs loss to the ex chequer.

o North-South Sub-corridor - Approved  The gov ernment has mandated that the
for preparing DPRs International Mobile Equipment Identity
(IMEI) number (1 5 -digit unique numeric
o Southern DFC - Y et to be approved. identifier) be disclosed prior to the sale of
mobile dev ices in India.
 Besides a world-class infrastructure, DFC also
ensures:  The mobile networks will hav e access to the list
of approv ed IMEI numbers and entry of any
o High energy efficiency unauthorised mobile phones on their network.
o Reduced unit logistic cost

o Faster transit
16-05-2023
o Employ ment generation 1 ) Consider the following statements with respect to
Permafrost
o Real estate ex pansion
1. Permafrost is any ground that remains
o Env ironment-friendly operations completely frozen (0°C) or colder for at least 2
y ears straight.
Double-stack savings
2. Permafrost can be found both on land and
 The av erage speed of goods trains in India is 25
below the ocean floor.
kmph, while on DFC route it is more than
double at 60 kmph. 3. Permafrost is found only in the arctic regions.
Dedicated Freight Corridor Corporation of Which of the statement(s) giv en above is/are correct?
India Limited (DFCCIL)
a. 2 only
 It is a public sector undertaking which
undertakes planning, dev elopment, and b. 2 and 3 only
mobilisation of financial resources of the
Dedicated Freight Corridors. c. 1 and 2 only

 It has been designated by the Gov ernment of d. 1 , 2 and 3


India as a Ministry of Railway s Enterprise.

5. d
2) Consider the following statements with respect to
The CEIR system, is scheduled for pan-India launch. Report on International Religious Freedom

Central Equipment Identity Register (CEIR) 1. It is an annual report released by The


Sy stem Committee of Religious NGOs, United Nations
Organ.
 It is to ease reporting of stolen and lost mobiles
and block the use of mobiles all ov er the 2. The committee promotes religious freedom for
country. the UN member states and observer countries.

 It enable the tracing of the stolen and lost 3. The report recommended for designating India
mobiles to the police, detection of cloned or as a Country of Particular Concern (CPC).
counterfeit mobiles.
Which of the statement(s) giv en above is/are correct?
 It will restrict the use of such cloned mobiles, as
a. 2 and 3 only
well as protect the interest of the consumers by
making them aware of the information related b. 1 and 2 only
to fake and cloned mobile phones.
c. 3 only
 The CEIR sy stem is equipped with an in-built
mechanism that allows it to detect the use of d. 1 , 2 and 3
cloned mobile phones across all te lecom
networks.

CHENNAI |SALEM| MADURAI | COIMBATORE DELHI | BANGALORE | T HIRUVANANTHAPURAM


www.shankariasacademy.com 38
www.iasparliament.com

3) Consider the following statements with respect to Answers


Carbon Capture and Storage
1. c
1. Carbon capture and storage (CCS) is the
capture and storing of carbon dioxide (CO2) Thaw ing permafrost in the Arctic could unlock toxic
and Carbon-monoxide (CO) before it is released w aste buried for decades.
into the atmosphere.
Perm afrost
2. The captured carbon can be stored only in
geological process.  Permafrost is any ground that remains
completely frozen 32°F (0°C) or colder for at
Which of the statement(s) giv en above is/are correct? least 2 y ears straight.

a. 1 only  These permanently frozen grounds are most


common in regions with high mountains and in
b. 2 only Earth’s higher latitudes near the North and
South Poles.
c. Both 1 and 2
 Permafrost can be found on land and below the
d. Neither 1 nor 2 ocean floor.

 It is often found in Arctic regions such as


Greenland, the U.S. state of Alaska, Russia,
4) Consider the following statements with respect to China, and Eastern Europe.
Energy Transitions Working Group (ETWG)
 It is found in mountainous regions such as the
1. It is a working group under G-7 forum.
South American Andes and New Zealand's
2. International Energy Agency (IEA) and United Southern Alps, as well as below Antarctica.
Nations Env ironment Program (UNEP)
 Permafrost is made of a combination of soil,
participated in the 3rd ETWG meeting.
rocks and sand that are held together by ice.
Which of the statement(s) giv en above is/are correct?
 Near the surface, permafrost soils contain large
a. 1 only quantities of organic carbon leftover from dead
plants that couldn’t decompose due to the cold.
b. 2 only
 A lay er of soil on top of permafrost does not stay
c. Both 1 and 2 frozen all y ear.

d. Neither 1 nor 2  This lay er, called the active layer, thaws during
the warm summer months and freezes again in
the fall.

5) Consider the following statements with respect to  Permafrost does not alway s form in one solid
Union Public Serv ice Commission (UPSC) sheet – it is both continuous and discontinuous.
1. UPSC recruits both Group A and Group B Im pacts
central services of India.
 Increase in water levels
2. The UPSC consists of a chairman and other
members appointed by the President of India.  Increased erosion
3. The chairman of the Commission hold office for  Traces of zombie v irus that can infect humans
a term of 6 y ears or until they attain the age of hav e been found preserved in permafrost.
62 y ears.
 Co2 and methane released from permafrost
Which of the statement(s) giv en above is/are correct? could cause global warming.
a. 2 only
2. c
b. 2 and 3 only Report on International Religious Freedom
c. 1 and 2 only
 It is an annual Report released by the US
d. 1 , 2 and 3 Commission on International Religious
Freedom (USCIRF).

CHENNAI |SALEM| MADURAI | COIMBATORE DELHI | BANGALORE | T HIRUVANANTHAPURAM


www.shankariasacademy.com 39
www.iasparliament.com

 The US Congressional-mandatory annual Industrial Carbon Sequestration


report on international religious freedom
documents the status of religious freedom in  This is not a widely renowned method, but it
countries across the world. can be used in some industries.

 India is recommended to be designated as a  They capture the carbon in 3 way s from a power
Country of Particular Concern for the 4th time plant, pre-combustion, post-combustion and
in a row. ox y-fuel.

 The ‘Country of Particular Concern’ (CPC) is the 4. b


category of countries whose governments either
engage in or tolerate “particularly severe The 3rd Energy Transitions Working Group Meeting
v iolations” of religious freedom. (ETWG) is being held under India’s G20 Presidency, in
Mumbai.
United States Com mission on International
Religious Freedom (USCIRF) Energy Transitions Working Group (ETWG)

 It is an independent bi-partisan US federal  It is a grouping that focuses on achieving energy


gov ernment commission that was created by transition in the world.
the International Religious Freedom Act
(IRFA) 1 998.  The 3-day ’s meeting has participation of over
1 00 delegates from G20 member countries,
 It has a mandate to monitor religious freedom special inv itee countries.
v iolations globally and make policy
recommendations to the U.S. President, the  International Organisations such as
Secretary of State, and the Congress. International Energy Agency (IEA), and United
Nations Env ironment Program (UNEP) also
 It is a congressionally created entity and not an participated.
NGO or adv ocacy organization.
 It discussed the need to giv e preference to the
 The report recommendations are v oluntary in dev elopment and adoption of sustainable
nature. energy sources and policies that promote
conservation and reduce carbon emissions.
3. d
 The Meeting was complemented by 3 side
Carbon Capture and Storage (CCS) ev ents.

 It is a way of reducing carbon emissions, which  Workshop with Multilateral Dev elopment
could be key to helping to tackle global Banks (MDBs)
warming.
 Seminar on Just Transition Roadmap
T y pes of Carbon Sequestration
 Seminar on Biofuels
Biological Carbon Sequestration
5. c
 This roughly is the storage of carbon dioxide in
v egetation like grasslands and forests, as wellas Union Public Serv ice Commission (UPSC)
in soils and oceans.
 It is the central recruiting agency in India.
Geological Carbon Sequestration
 UPSC recruits both Group A and Group B
 This is where carbon diox ide is stored in (EPFO) central services of India.
underground geologic formations, such as in
rocks.  It is an independent constitutional body in the
sense that it has been directly created by the
T echnological Carbon Sequestration Constitution.

 This is a relativ ely new way of capturing and  Articles 31 5 to 323 in Part XIV of the
storing carbon diox ide and continues to be Constitution contain elaborate prov isions
ex plored by scientists. regarding the UPSC.

 The method looking into more way s of using  The UPSC consists of a chairman and other
carbon diox ide as a resource rather than members appointed by the President of India.
removing it from the atmosphere.
 The Constitution, without specifying the
strength of the Commission has left the matter

CHENNAI |SALEM| MADURAI | COIMBATORE DELHI | BANGALORE | T HIRUVANANTHAPURAM


www.shankariasacademy.com 40
www.iasparliament.com

to the discretion of the president, who 3. The Chin people of My anmar and Mizo people
determines its composition. of Mizoram are a group of related indiv iduals
with respect to Kuki people.
 Usually , the Commission consists of nine to
elev en members including the chairman. Which of the statement(s) giv en above is/are correct?

 The chairman and members of the Commission a. 2 only


hold office for a term of 6 y ears or until they
attain the age of 65 y ears, whichever is earlier. b. 3 only

 Howev er, they can relinquish their offices at c. 2 and 3 only


any time by addressing their resignation to the
d. 1 , 2 and 3
president.

 They can also be removed before the expiry of


their term by the president in the manner as 3) Consider the following statements with respect to
prov ided in the Constitution. Liquors
 It conducts ex aminations for appointments to 1. Liquor is an alcoholic beverage mainly made up
the all-India serv ices, Central serv ices and of methanol.
public serv ices of the centrally administered
territories. 2. Spurious liquor is characterized by the liquid
mix ture containing methanol as well as
 The role of UPSC is not only limited, but also Ethanol.
recommendations made by it are only of
adv isory nature and hence, not binding on the 3. Food Safety and Standards (Alcoholic
gov ernment. Bev erages) Regulations, 201 8 stipulate the
max imum permissible quantity of methanol in
different liquors.

17-05-2023 Which of the statement(s) giv en above is/are correct?


1 ) Consider the following statements with respect to Na- a. 1 and 2 only
ion Batteries
b. 3 only
1. Na-ion batteries are energy dense, non-
flammable, and can operate well in cold c. 2 and 3 only
temperatures.
d. 1 , 2 and 3
2. Unlike Li-ion battery, Na-ion is a ty pe of non-
rechargeable battery.

3. Lithium (Li) is heav ier and possesses a lower


4) Consider the following statements with respect to
standard electrochemical potential than
Sodium (Na). Calcium-41

Which of the statement(s) giv en above is/are correct?


1. Calcium-41 is a rare long-lived radio-isotope of
a. 1 only Calcium.

b. 1 and 2 only 2. It is used in radiometric dating to determine the


age of objects older than 50,000 y ears.
c. 2 and 3 only
3. Calcium-41 ’s half-life is lower than the Carbon-
d. 1 , 2 and 3 1 4 which is used in conventional carbon dating
method.

Which of the statement(s) giv en above is/are correct?


2) Consider the following statements with respect to
Kuki Tribes a. 1 only

1. Kuki Tribes are confined only to the North b. 2 and 3 only


eastern states of India.
c. 1 and 2 only
2. Kuki Tribes are designated as Particularly
V ulnerable Tribal Groups (PV TGs) in Manipur. d. 1 , 2 and 3

CHENNAI |SALEM| MADURAI | COIMBATORE DELHI | BANGALORE | T HIRUVANANTHAPURAM


www.shankariasacademy.com 41
www.iasparliament.com

5) Consider the following statements with respect to Na-ion Li-ion


Trachoma
 It is more abundant  It is less abundant
1. It is a chronic viral infection that may even lead
to blindness.  It is cheaper  It is costlier

2. WHO recommended SAFE strategy to  Less reactive  Highly reactive


eliminate Trachoma.
 High energy density  Low energy density
3. India has been granted trachoma free status by
WHO in 2022.  Longer battery life  Smaller battery life
Which of the statement(s) giv en above is/are correct?  Longer life-span  Shorter life-span
a. 2 only
 More eco-friendly  Less eco-friendly
b. 1 and 2 only
2. b
c. 1 and 3 only
Kuki Tribes demanded for a separate Kukiland
d. 1 , 2 and 3
Kuki T ribes

 The Kuki ethnic group, which originated in the


Answers Mizo hills, includes the Chins of My anmar, the
Mizos of Mizoram, and the Kukis of
1. a Bangladesh.

Scientists have found an alternative environment-  The three are collectively known as Zo people.
friendly Na-ion batteries for Li-batteries
 Kuki Tribes are believed to be the native people
Na-ion Batteries of Mizoram.
 The sodium-ion (Na-ion) battery is a ty pe of  Kuki tribes inhabited in the regions of
rechargeable battery that uses sodium ions My anmar, Manipur, Assam and Mizoram of
(Na+) as its charge carriers. India and Bangladesh.
 Na-ion batteries are energy dense, non-  Around 50 tribes of kuki people in India are
flammable, and can operate well in cold recognized as scheduled tribes based on dialect
temperatures. spoken and region of origin.
 Sodium-ion batteries can use aqueous as wellas  Kuki Tribes are mainly followers of
non-aqueous electrolytes. Christianity .
Na-ion v s Li-ion  Traditionally the Kuki liv ed in small
settlements in the jungles, each ruled by its own
 Na is thrice heav ier and possesses a lower
chief called Lal.
standard electrochemical potential than Li.
 Kuki Tribes are not designated as Particularly
 Both ty pes of batteries use a liquid electrolyte to V ulnerable Tribal Groups (PV TGs) in Manipur.
store and transfer electrical energy, but differ in
the ty pe of ions they use. 3. c
 Na-ion battery is Cost-effective and using raw 22 people had died after consuming spurious liquor in
materials that is abundant from earth Chengalpattu and V illupuram districts of Tamil Nadu
compared to Li-ion batteries.
Liquors
 Both ty pes of batteries are rechargeable.
 Ethanol is a clear, colorless liquid.

 It is also called ethy l alcohol, grain alcohol,


drinking alcohol, is an organic compound.

 Liquor is differentiated by its alcohol content


from the 5% or so of beer to the 1 2% or so of
wine to the 40% or so of distilled spirits (all by
v olume).

CHENNAI |SALEM| MADURAI | COIMBATORE DELHI | BANGALORE | T HIRUVANANTHAPURAM


www.shankariasacademy.com 42
www.iasparliament.com

 Ethanol is a psy choactive drug in low doses,  C-1 4 uses the carbon content in an organic
reduces the lev el of neurotransmission in the matter to determine the age and Ca-41 uses the
body , leading to its ty pical intoxicating effects. calcium atoms to determine the age when
ex posed to cosmic rays.
 Spurious liquor – It is characterised by the
liquid mix ture containing methanol as well as 5. a
Ethanol.
Benin, Mali in Africa defeat trachoma as public health
 The Food Safety and Standards (Alcoholic problem
Bev erages) Regulations 201 8 stipulate the
max imum permissible quantity of methanol in T rachoma
different liquors.
 Trachoma is a disease of the ey e caused by
Methanol infection with the bacterium Chlamydia
trachomatis.
 Hazardous Chemical Rules 1 989 includes
methanol as a hazardous one.  In its early stages, trachoma causes
conjunctivitis (pink ey e).
 The accumulation of formic acid in methonal
ov er time leads to a baneful condition called  Blindness from trachoma is irreversible.
metabolic acidosis.
 Trachoma infection is transmitted by direct or
4. c indirect transfer of ey e and nose discharges of
infected people.
A new study shows a way to use calcium-41 the same
w ay carbon-14 has been used in c arbon-dating, but  It particularly affects y oung children, who
w ith several advantages. harbour the principal reservoir of infection.

Radiom etric Dating India’s status

 When an organic entity is aliv e, its body keeps  According to WHO, the prev alence of trachoma
absorbing and losing carbon-14 atoms. should be less than 5% to mark it as eliminated.

 When it dies, this process stops and the extant  In its initial study , AIIMS has noted the
carbon-14 starts to decay away. prev alence of around 3.5% in India.

 Using the difference between the relative  WHO is y et to declare India trachoma free.
abundance of these atoms in the body and the
number that should’ve been there, researchers World Health Organization on Trachoma
can estimate when the entity died.
 WHO-recommended SAFE strategy to
 Carbon-1 4 has a half-life of 5,7 30 y ears. eliminate trachoma. It includes

 Carbon-1 4 atoms occur once in around 1 0-12 o Surgery to prev ent blindness in those
carbon atoms. who hav e trichiasis/entropion.

Ca-41 o Antibiotics (tetracycline ointment or


azithromycin) to combat active
 Calcium-41 has a half-life of 99,400 y ears. chlamy dial infection.

 It's produced when cosmic ray s from space o Facial hy giene.


smash into calcium atoms in the soil, and is
found in the earth's crust, opening the door to o Env ironmental change.
dating fossilised bones and rock.

 Calcium-41 is rarer, occurring once in around


1 0-1 5 calcium atoms. 18-05-2023
1 ) Consider the following statements with respect to the
C-14 v s Ca-41 Trimbakeshwar Temple:

 Calcium-41 ’s half-life (99,400 y ears) is higher 1. It was constructed by third Peshwa Balaji
than the carbon-14’s half-life (5,7 30 y ears) Bajirao and consists one of the twelve
which is used in conv entional carbon dating Jy otirlingas.
method.

CHENNAI |SALEM| MADURAI | COIMBATORE DELHI | BANGALORE | T HIRUVANANTHAPURAM


www.shankariasacademy.com 43
www.iasparliament.com

2. It is located on the eastern banks of River 4) Consider the following statements with respect to
Mahanadi, in the state of Odisha. London Interbank Offered Rate (LIBOR):

3. The temple is part of the UNESCO intangible 1. It is a global interest rate at which major global
heritage list. banks lend to one another in the international
interbank market for short-term loans.
Which of the abov e statement(s) is/are correct?
2. The interest rate is calculated only for
a. 1 only currencies such as the Euro, US dollars, Swiss
Franc, Japanese Yen and UK Pound Sterling.
b. 1 and 2 only
3. India has its own benchmark interest rate called
c. 3 only the Secured Ov ernight Financing Rate (SOFR).
d. 2 and 3 only Which of the abov e statement(s) is/are correct?

a. 1 and 3 only
2) The gov ernment has dev eloped an AI & Facial b. 1 and 2 only
Recognition powered Solution for Telecom SIM
Subscriber V erification (ASTR), to bring down cyber c. 2 and 3 only
frauds. Consider the following statements with respect
to ASTR: d. 1 , 2 and 3

1. It is dev eloped by the department of


communications, Ministry of Science and
Technology. 5) Consider the following statements:

2. Telangana is using the ASTR feature to v erify 1. Dengue is transmitted by the Aedes mosquito
the beneficiaries of public distribution system species, which is also responsible for the spread
(PDS). of Chikunguny a and Zika v irus.

Which of the abov e statement(s) is/are correct? 2. India has its indigenous recombinant type of
v accine for dengue.
a. 1 only
3. The recombinant v accine is similar to Cov axin,
b. 2 only which uses the non-structural proteins of the
dengue v irus.
c. Both 1 and 2
Which of the abov e statement(s) is/are incorrect?
d. Neither 1 nor 2
a. 1 and 2 only

b. 1 and 3 only
3) Consider the following statements with respect to the
Rasht-Astara Railway Project: c. 2 and 3 only

1. It is part of the International North-South d. 1 , 2 and 3


Transport Corridor (INSTC).

2. The project passes through India, Iran,


Azerbaijan, and Russia. Answers
3. The project connects the Baltic Sea, Caspian 1. a
Sea, Indian Ocean and the Black Sea.
T rimbakeshwar T emple
Which of the abov e statement(s) is/are incorrect?
 It is located in the state of Maharashtra.
a. 1 only
 It is located near the mountain named
b. 2 only Brahamagiri from which the riv er Godavari
flows.
c. 2 and 3 only
 It was constructed by third Peshwa Balaji
d. 1 , 2 and 3 Bajirao (1 7 40-1760) on the site of an old temple.

CHENNAI |SALEM| MADURAI | COIMBATORE DELHI | BANGALORE | T HIRUVANANTHAPURAM


www.shankariasacademy.com 44
www.iasparliament.com

 Trimbakeshwar Temple is a religious centre Iran, Russia, Azerbaijan and other countries via
hav ing one of the twelv e Jy otirlingas. railway s and sea.

 It is not part of the UNESCO intangible heritage  The project is part of the International North-
list. South Transport Corridor (INSTC).

2. d  The 1 62km railway along the Caspian Sea coast


would help to co nnect Russian ports on the
Artificial Intelligence and Facial Recognition Baltic Sea with Iranian ports in the Indian
powered Solution for T elecom SIM Subscriber Ocean and the Gulf.
Verification (ASTR)

 The Department of Telecommunications


(DoT), the Ministry of Communications, has
dev eloped an artificial-intelligence-based facial
recognition tool.

 The tool has the capability of running checks on


subscriber databases of telecom operators to
deduce whether it contains multiple
connections associated with the same person.

Working of AST R

 Human face images are encoded using


conv olutional neural network (CNN) models in 4. b
order to account for the tilt and angle of the
face, opaqueness and dark colour of the images. London Interbank Offered Rate (LIBOR)

 After that, a face comparison is carried out for  LIBOR is a global benchmark interest rate that
each face against all faces in the database, and combines individual rates at which banks opine
similar faces are grouped under one directory. they may borrow from each o ther at the London
interbank market.
 Two faces are concluded to be identical by
ASTR if they match to the ex tent of at least  In other words, it is the benchmark interest rate
97 .5%. at which major global banks lend to one another
in the international interbank market for short-
 ASTR is capable of detecting all SIMs against a term loans.
suspected face in less than 1 0 seconds from a
database of 1 crore images.  It is used as a benchmark to settle trades in
futures, options, swaps and other derivative
 The DoT allows an indiv idual to take nine financial instruments in ov er-the-counter
legitimate mobile phone connections using a markets and on ex changes globally.
single identity proof.
 Further, consumer lending products including
 In essence, what the ASTR does is: mortgages, credit cards and student loans,
among others, too use it as a benchmark rate.
1. It looks up if there are more than nine
connections against a single  LIBOR is calculated for fiv e currencies such as
indiv idual’s photograph; the U.S. dollar, Euro, Pound, Swiss Franc and
Japanese Y en.
2. It runs a search through the database to
see if the same person has taken SIMs
 The Secured Ov ernight Financing Rate (SOFR)
under different names. is a benchmark interest rate for dollar-
3. c denominated deriv atives and loans that will
replace LIBOR.
Rasht-Astara Railway Project
5. c
 Russia and Iran sign deal for railway corridor
Dengue
intended to riv al Suez Canal.
 Dengue is transmitted to humans by the Aedes
 Rasht-Astara railway is seen as an important
mosquito species, Ae. Aegypti or Ae.
link in the corridor, meant to connect India,
Albopictus, which also spreads Chikungunya
and Zika v irus.

CHENNAI |SALEM| MADURAI | COIMBATORE DELHI | BANGALORE | T HIRUVANANTHAPURAM


www.shankariasacademy.com 45
www.iasparliament.com

 India does not hav e an indigenous v accine for a. 1 only


dengue.
b. 2 and 3 only
 There are four serotypes (or ty pes) of the
dengue v irus, DEN-1 , DEN-2, DEN-3 and DEN- c. 1 and 2 only
4, each v irus interacting differently with
antibodies in the human body. d. 1 , 2 and 3

 Each serotype is capable of manifesting into


dengue fev er, dengue haemorrhagic fev er and 3) Consider the following statements with respect to
dengue shock sy ndrome. Gharials
 Fiv e ty pes of dengue v accines are currently 1. They liv e in shallow clear freshwater river
being inv estigated: sy stems and confined only to Chambal Riv er in
India.
1. Liv e attenuated v accine (which uses the
weakened or attenuated form of the v irus, such 2. Gharial Conserv ation Alliance (GCA) is an
as the measles or chickenpox vaccine); initiativ e under the Ministry of Env ironment,
forest and climate change.
2. Inactivated v accine (using the dead v irus, used
for Hepatitis A and rabies), Which of the statement(s) giv en above is/are correct?
3. Recombinant subunit v accine (as in a. 1 only
COV ISHIELD, where non-structural proteins
of the dengue v irus are used, aiding a balanced b. 2 only
immune response),
c. Both 1 and 2
4. V iral v ectored v accine (such as the v accine
against Ebola) d. Neither 1 nor 2

5. DNA v accine (for HIV , malaria, TB)

4) The place Neh Pema Shelpu Drupkhang recently


seen in news is located in which of the following state?
19-05-2023
a. Jammu & Kashmir
1 ) The term Dotted Lands recently seen in news is
related to? b. Arunachal Pradesh
a. The lands that were giv en as v oluntary gift from c. Manipur
the landlords to the landless.
d. Sikkim
b. It is a disputed land for which there are no clear
ownership documents.

c. It is a land donated by kings or wealthy patrons 5) Consider the following statements with respect to
to Brahmins. Spitzer Telescope

d. It is a land giv en to temples during 1. It is a Cassegrain reflector telescope that uses a


harshav ardhana period. combination of a primary concave mirror and a
secondary convex mirror.

2. It is the final mission in NASA's Great


2) Consider the statements with respect to G-7 and G- Observ atories Program.
20
3. Spitzer Resurrector Mission is an initiative of
1. India holds the Presidency of the G20 from NASA to restart the telescope to its full
2022 to 2023. efficiency.
2. The European Union and Russia are part of Which of the statement(s) giv en above is/are correct?
both G7 and G20 countries.
a. 2 only
3. G-7 and G-20 both has the permanent
secretariat in U.S. b. 2 and 3 only

Which of the statement(s) giv en above c. 1 and 2 only


is/are not correct?

CHENNAI |SALEM| MADURAI | COIMBATORE DELHI | BANGALORE | T HIRUVANANTHAPURAM


www.shankariasacademy.com 46
www.iasparliament.com

d. 1 , 2 and 3  G7 has no legal ex istence or official members.

G-20

Answers  It is the premier forum for international


economic cooperation.
1. b
 G20 stands for group of 20 members (19
Andhra Pradesh government has started removing
countries and Europen Union).
“dotted lands” in the state from the prohibited list.
 It holds annual summit under the leadership of
Dotted Lands
a rotating Presidency.
 Dotted lands are disputed lands for which there
are no clear ownership documents.  It represent around 85% of the global GDP, over
7 5% of the global trade, and about two -thirds of
 The farmers could not procure loans from the world population.
banks and financial institutions by putting up
the land as collateral.  The Group does not hav e a permanent
secretariat.
 Financial institutions do not recognize dotted
land documents as clear ownership documents.  India holds the Presidency of the G20 from
2022 to 2023
Andhra Pradesh’s Recent Amendment

 Andhra Pradesh government introduced a Bill


to amend the Rev enue Act to grant titles to
farmers who hav e been cultivating dotted lands
for more than 1 2 y ears.

 The dots, and entries in land registers, will be


removed and these farmers will be giv en clear
land ownership documents.

 The lands are now taken off from the prohibited


list, so landowners/farmers will get full rights
ov er the lands.

 More importantly, they can apply for financial


assistance for crop support, purchase seeds and
fertilisers, and procure farm equipment. 3. d

 The landowners/farmers can also sell the lands World Wildlife Fund-Pakistan (WWF-Pakistan)
or gift to kin or relatives. confirmed reports of gharials being present in Punjab

2. b Gharials (Gav ialis gangeticus)

External Affairs Minister S Jaishankar met Chinese  Status - Critically endangered.


Foreign Minister at the G20 Foreign Ministers Meeting
 It is also listed in Appendix I of CITES.
Com parison of G-7 with G-20
 It is also known as gav ial or fish-eating
G-7 crocodile, is a type of Asian crocodilian set apart
by its long and thin snout.
 G7 stands for an informal Group of Seven
countries.  Habitat - They liv e in clear freshwater river
sy stems, congregating at riv er bends where the
 G-7 Consists of Canada, France, Germany, water is deeper.
Italy , Japan, U.K and U.S.
 Once found from Pakistan to My anmar, the
 It holds an annual summit with European reptile's range has now shrunk to 2 countries-
Union and other inv itees. India (Chambal, Girwa, and Son Riv ers) and
Nepal (Naray ani Riv er).
 G7 includes 40% of global GDP and 1 0% of the
world’s population.  In India, the National Chambal Sanctuary holds
about 90% of the surv iving gharials.

CHENNAI |SALEM| MADURAI | COIMBATORE DELHI | BANGALORE | T HIRUVANANTHAPURAM


www.shankariasacademy.com 47
www.iasparliament.com

Conservation Efforts  It was launched in 2003 by NASA, U.S.

 In collaboration with the Uttar Pradesh Forest  It was comprised of two major components:
Department, WWF-India started a gharial
reintroduction programme at Hastinapur  T he Cry ogenic Telescope Assembly -
Wildlife Sanctuary. contained the 85 centimeter telescope and
Spitzer's three scientific instruments.
 Since January 2009, 250 captive reared gharial
from Kukrail Rehabilitation Centre (Lucknow)  T he Spacecraft - controlled the telescope,
hav e been released into Riv er Ganga. prov ided power to the instruments, handled the
scientific data and communicated with Earth.
 India’s Katerniaghat Wildlife Sanctuary
contains a breeding population v ital to the  It is the final mission in NASA's Great
surv ival of the species. Observ atories Program a family of 4 space-
based observatories each observing the
4. b Univ erse in a different kind of light.

A 2-member team of the National Commission for  The other missions in the program include the
Minorities (NCM) visited Mechukha in Arunachal v isible-light Hubble Space Telescope (HST),
Pradesh’s Shi-Y omi district Compton Gamma-Ray Observ atory (CGRO),
and the Chandra X-Ray Observ atory (CXO).
Neh Pem a Shelpu Drupkhang
 Spitzer continued to operate until 2020.
 It is located in Mechukha, Shi-Y omi district of
Arunachal Pradesh. Spitzer Resurrection Mission

 According to Buddhists, it is a Pilgrimage place,  The innov ation arm – SpaceWERX of the
known as the meditation place of 6th rebirth of United States Space Force selected a startup
Guru Padma Samba, one of the founding Rhea Space Activity, to dev elop the Spitzer
fathers of Tibetan Buddhism in 7 49 A.D. Resurrector Mission.
 According to Sikhism, it is a historic place  The mission aims to send a spacecraft to Spitzer
which is associated with Guru Nanak, the to serv ice and restore its operation.
founder of Sikhism.
 This will act as a demonstration for the In-space
 Recently, it was claimed to be converted into a Serv ice Assembly and Manufacturing (ISAM)
Buddhist shrine by the local Buddhist Memba techniques.
community.
 After its resurrection, the telescope will be used
Mem ba Community to detect and characterise the potentially
hazardous Near Earth Objects (NEOs) and for
 Memba tribal community is the third largest other astronomical observations.
tribal community among the six Buddhist tribes
of Arunachal Pradesh.

 Most of these tribal communities are scattered 20-05-2023


around the Eastern parts of Siang District,
Arunachal Pradesh. 1 ) Consider the following statements with respect to
Strengthening Teaching-Learning and Results for
 Poly gamy is prev alent where mainly women States (STARS) Program
can hav e more than one husband.
1. It aims to strengthen V ocational Education and
5. a School-to-Work Transition.

The Spitzer Resurrection Mission will send a 2. It is a central sector scheme aligned with
telerobotic space probe to bring the Spitzer space objectives of National Education Policy (NEP)
telescope back to life. 2020.

Spitzer T elescope 3. It is a World Bank aided project under the


Ministry of Education.
 Spitzer was designed to detect infrared
radiation, which is primarily heat radiation. Which of the statement(s) giv en above is/are correct?

 It allows to peer into regions of space that are a. 1 only


hidden from optical telescopes.
b. 1 and 3 only

CHENNAI |SALEM| MADURAI | COIMBATORE DELHI | BANGALORE | T HIRUVANANTHAPURAM


www.shankariasacademy.com 48
www.iasparliament.com

c. 2 and 3 only 3. Dead Sea - Eastern


Asia
d. 1 , 2 and 3
4. Toshka
Lakes - Northeastern
Africa
2) Consider the following statements with respect to
Open Network for Digital Commerce (ONDC) Select the correct answer using the codes given below:

1. It is a not-for-profit company incorporated a. 1 and 2 only


under Section 8 of the Companies Act 2013.
b. 2, 3 and 4 only
2. It aims to promote open networks for all aspects
of ex change of goods and serv ices over digital c. 1 and 4 only
networks.
d. 1 , 2, 3 and 4
3. Unlike the current platform centric model, it
works based on network centric model.

Which of the statement(s) giv en above is/are correct? 5) Operation Karuna recently seen in news is related
to?
a. 1 only
a. It is to curb the cy ber-enabled crime networks
b. 1 and 3 only across India.

c. 2 and 3 only b. Indian Mission to assist My anmar which has


been dev astated by Cy clone Mocha.
d. 1 , 2 and 3
c. To suppress the Kuki and Meitei tribes from
v iolence in Manipur.

3) Under the recent RBI Framework for green deposits, d. To combat the proliferating illegal trade of live
which of the following sectors are eligible for Green turtles and tortoises.
Deposits?

1. Renewable Energy
Answers
2. Sustainable Water
1. b
3. Nuclear power
Ministry of Education and World Bank organized a
4. Pollution Prevention and Control w orkshop on School-to-Work Transition under the
STARS Program in Mumbai.
5. Fossil fuels
Strengthening Teaching-Learning and Results
Select the correct answer using the codes given below: for States (ST ARS) Program

a. 1 and 3 only  Aim – To strengthen V ocational Education and


School-to-Work Transition.
b. 1 , 2 and 4 only
 Agency – Ministry of Education and World
c. 1 , 2, 4 and 5 only Bank.
d. 1 , 2, 3, 4 and 5
 It is a centrally sponsored scheme.

 It is carved out of Samagra Shiksha, with a focus


4) Match the following Dry ing Lakes with their on those elements of the scheme that will most
respective Regions directly support school education
enhancement.
Drying Lakes Regions
 6 ST ARS States - Himachal Pradesh, Kerala,
1. Aral Sea - Central Odisha, Rajasthan, Madhy a Pradesh,
Asia Maharashtra

2. Xinkai Lake - Middle  A broad-based strategy was proposed with


East focus on

CHENNAI |SALEM| MADURAI | COIMBATORE DELHI | BANGALORE | T HIRUVANANTHAPURAM


www.shankariasacademy.com 49
www.iasparliament.com

o V ocational education,  The platform will also be compliant with the


Information Technology Act, 2000.
o Industry tie-ups,
3. b
o Integrating vocational studies with
school curricula and Reserve Bank of India (RBI) came up w ith a regulatory
framew ork for banks to accept green deposits from
o Modify ing the ex isting school sy llabus customers
to meet industry demands and make
students job ready after class 12th. Green Deposits

 The ov erall focus and components of the STARS  A green deposit is a fix ed-term deposit for
project are aligned with objectives of National inv estors looking to inv est in env ironmentally
Education Policy (NEP) 2020. friendly projects.

 The program supports the National Assessment  Green deposits are v ary from the regular
Centre, PARAKH deposits only on that banks promise to earmark
the money to be used towards env ironment-
PARAKH friendly projects.

 Performance Assessment, Rev iew, and Analysis  Banks will hav e to come up with a set of rules or
of Knowledge for Holistic Dev elopment policies approved by their respective Boards
(PARAKH). that need to be followed while inv esting green
deposits from customers.
 It has been launched as part of implementing
National Education Policy (NEP) 2020. Sectors eligible for green deposits

 Aim - Build students confidence and motivate  Renewable Energy ,


them to make progress in personal and
professional career.  Clean Transportation,

 Dev elop new learning programs that matches  Pollution Prevention and Control,
with the student's interest and their skill sets.
 Green Building,
2. d
 Sustainable Water,
Open Network for Digital Commerce (ONDC)
 Wastewater Management, and others.
 It is a not-for-profit company inc orporated
under Section 8 of the Companies Act 2013, Sectors ineligible for green deposits
manages and operates the ONDC Network.
 Fossil fuels,
 Aim - It aims to promote open networks for all
aspects of ex change of goods and services over  Nuclear power,
digital networks.
 Tobacco, etc,.
 Unlike current platform centric model, it works
based on network centric model.  Banks - HSBC and HDFC hav e launched green
deposits in India for corporates as well as
 It is based on open-sourced methodology indiv iduals.
independent of any specific platform.
4. c
 It aims to create a lev el play ing field for e -
commerce behemoths such as Amazon, A recent study shows that more than half of the w orld's
Flipkart, and offline traders. large lakes are drying up.

 It is to create new opportunities, curb digital Aral Sea


monopolies and by supporting MSME and
small traders and help them get on online  It is a saltwater lake located in Central Asia.
platforms.
 It shares the boundary between Kazakhstan to
 Agency - It is an initiativ e of the Department the north and Uzbekistan to the south.
for Promotion of Industry and Internal Trade
(DPIIT) under the Ministry of Commerce and  It has shrinked in its area and v olume that
Industry. began in the second half of the 20th century.

CHENNAI |SALEM| MADURAI | COIMBATORE DELHI | BANGALORE | T HIRUVANANTHAPURAM


www.shankariasacademy.com 50
www.iasparliament.com

Dead Sea  The India Meteorological Department (IMD)


categorised Cy cloe Mocha as an ‘Extremely
 It is a salt lake between Israel and Jordan in Severe Cyclonic Storm’.
southwestern Asia.
 Y emen suggested the name Mocha supposed to
 Jordan Riv er is the main source of water for this be pronounced as Mokha.
lake.

 It div erted its flow from mid-2010 so the lake is


getting drier. 21-05-2023
1 ) Consider the following statements with respect to
World Health Assembly

1. It is the apex decision making body of World


Health Organization (WHO).

2. The Health Assembly is held biennially in


Genev a, Switzerland.

Which of the statement(s) giv en above is/are correct?

a. 1 only

b. 2 only

c. Both 1 and 2

Xinkai Lake d. Neither 1 nor 2

 It is located in Eastern Asia.

 It is a freshwater lake is located in northern 2) Consider the following statements with respect to
Inner Mongolia near the Russian border. Rev iew Petitions

T oshka Lakes 1. High courts doesn’t has the power to review any
judgement or made by it as the Supreme Court
 It is located in north–eastern corner of Africa, a does.
hy per arid area in the Western Egy ptian Desert.
2. Only parties to a case can file a review petition
 This lake is diminishing with low Nile in Supreme Court.
discharges and appeared again with the Lake
Nasser high flood. 3. Usually , a rev iew petition is heard after a
curative petition is dismissed by the judiciary.
5. b
Which of the statement(s) giv en above
India launches Operation Karuna to assist cyclone-hit is/are not correct?
Myanmar
a. 2 only
Operation Karuna
b. 1 and 2 only
 It is an Indian Mission to assist Myanmar which
has been dev astated by Cy clone Mocha. c. 2 and 3 only

d. 1 , 2 and 3
 It is an operation done by Indian Navy.

 Materials were brought by INS Shiv alik, INS


Kamorta and INS Sav itri and INS Gharial. 3) North-South Transport Corridor sometimes seen in
news is connecting which of the following seas?
Cy clone Mocha
a. Baltic Sea and Arabian Sea
 It formed ov er the southeast Bay of Bengal,
intensified into a v ery severe cyclonic storm. b. Red sea and Arabian Sea

 It made landfall on the My anmar coast near c. Gulf of Aden and Bay of Bengal
Sittwe and Cox ’s Bazar (Bangladesh).
d. Mediterranean Sea and Red Sea

CHENNAI |SALEM| MADURAI | COIMBATORE DELHI | BANGALORE | T HIRUVANANTHAPURAM


www.shankariasacademy.com 51
www.iasparliament.com

4) Consider the following statements with respect to  Headquarters - Genev a, Switzerland.


Ganga Praharis
 The health assembly is held annually in the
1. Ganga Praharis are primarily v olunteers from headquarters.
among the local communities selected by the
National Mission for Clean Ganga.  Member countries focuses on a specific health
agenda prepared by the Ex ecutive Board.
2. They are spread over the Ganga basin states of
Uttarakhand, Uttar Pradesh, Jharkhand,  It prov ides a baseline data on health issues and
Chattisgarh and West Bengal. helps in collaboration, adv ocate and connect
between its member states and the participants
3. These Praharis hav e to be above 21 y ears of age of the assembly.
on the date of joining the project and preferably
from the riv er side v illages.  The theme of this y ear’s Health Assembly is:
WHO at 7 5: Sav ing liv es, driving health for all.
4. They play a crucial role in rescuing of wildlife in
distress and reporting of illegal activ ities like Functions
poaching.
 To determine the policies of the Organization
Which of the statement(s) giv en above is/are correct?
 Appoint the Director-General
a. 1 , 2 and 3 only
 Superv ise financial policies, and
b. 1 and 4 only

c. 2 and 3 only  Rev iew and approve the proposed program


budget.
d. 1 , 2 and 3
2. d

Centre files review petition against SC order on Delhi


5) Consider the following statements with respect to services
Methane
Article 137 – Rev iew Petition
1. It is the primary component of natural gas and
biogas.  The Supreme Court shall hav e power to review
any judgment pronounced or order made by it
2. Methane is an abundant and short-lived Green under Article 1 37.
House Gas (GHG) compared to carbon dioxide.
 Rev iew of judgments or orders by the Supreme
3. It has more global warming potential compared Court is subject to the prov isions of any law
to carbon dioxide (Co2). made by Parliament or any rules made under
Article 1 45.
Which of the statement(s) giv en above are correct?
 It is not necessary that only parties to a case can
a. 1 and 2 only seek a rev iew of the judgment on it.

b. 1 and 3 only  As per the Civ il Procedure Code and the


Supreme Court Rules, any person aggrieved by
c. 2 and 3 only a ruling can seek a rev iew.
d. 1 , 2 and 3  It is the discretion of the Supreme Court to
entertain any review petition.

 The Supreme Court itself laid down 3 grounds


Answers for seeking a rev iew of a v erdict it has delivered.
1. a
o The discovery of new and important
Prime Minister attends 7 6th Session of the World matter or ev idence which, after the
Health Assembly in Geneva, Switzerland ex ercise of due diligence, was not
within the knowledge of the petitioner
World Health Assembly or could not be produced by him;

 The World Health Assembly is the decision- o Mistake or error apparent on the face of
making body of WHO which is attended by the record;
delegations from all 1 94 WHO Member States.
o Or any other sufficient reason

CHENNAI |SALEM| MADURAI | COIMBATORE DELHI | BANGALORE | T HIRUVANANTHAPURAM


www.shankariasacademy.com 52
www.iasparliament.com

 As per Supreme Court rules, 1 966, review 4. b


petition is to be filed within 30 day s of the
pronouncement of judgment or order. Ganga Prahari

 If a rev iew petition is dismissed by the Supreme  Ganga Praharis (Guardians of the Ganga) are
Court, it may consider a curative petition filed primarily v olunteers from local communities
by the petitioner so as to prev ent abuse of selected by the National Mission for Clean
process. Ganga.

 Curativ e Petition is the final option to approach  Project – It is under National Mission for
when the rev iew petition is dismissed by the Clean Ganga- Wildlife Institute of India
Supreme Court. (NMCG-WII) project ‘Biodiv ersity
Conserv ation and Ganga Rejuv enation’.
 Curativ e Petition is guaranteed under Article
1 37 of the Constitution of India.  Aim - They conserve Ganga Riv er with the
objective of aquatic life by maintaining the
 As per article 227 , High co urt as a court of riv er’s Aviral Dhara (Continuous Flow), Nirmal
record can review its own judgement or order Dhara (Unpolluted Flow).
made by it.
 They were spread over the Ganga basin states of
3.a Uttarakhand, Uttar Pradesh, Jharkhand, Bihar
and West Bengal.
North-South Transport Corridor
 These ‘Praharis’ have to be above 18 years of age
 It links Russia’s Baltic Sea coast to India’s on the date of joining the project and preferably
western ports in the Arabian Sea v ia Iran. from the riv er side v illages.

 The corridor includes rail, road, sea and river  Children below 1 8 y ears of age are enrolled
transport infrastructure. under the Bal Ganga Praharis programme.

 Mem ber Countries - This agreement was  They are also inv olved in activ ities such as
ratified by 1 3 countries — India, Russia, Iran, Shram Daan, Ghat Cleaning, Awareness
Azerbaijan, Belarus, Bulgaria, Armenia, campaign, Plantation Driv es and Coordination
Kazakhstan, Ky rgy zstan, Oman, Tajikistan, with local administration.
Turkey and Ukraine.
5. b
 There are three main corridor branches:
Methane (CH4)
o West, along the western shore of the
Caspian Sea through Russia and  Methane (CH4) is a hy drocarbon that is a
Azerbaijan; primary component of natural gas.

o East, along the eastern shore of the  Biogas contains roughly 50-7 0% methane, 30-
Caspian Sea through Kazakhstan and 40% carbon diox ide, and trace amounts of
Turkmenistan; other gases.

o Trans-Caspian, inv olving ferry and  Methane is the second most abundant
container lines on the Caspian Sea. anthropogenic greenhouse gas (GHG), after
carbon diox ide (CO2), accounting for about
20% of global emissions.

 Methane is both powerful (25 times as potent as


CO2) and short-lived compared to carbon
diox ide (Co2).

 It is emitted from a v ariety of anthropogenic


(human-influenced) and natural sources.

CHENNAI |SALEM| MADURAI | COIMBATORE DELHI | BANGALORE | T HIRUVANANTHAPURAM


www.shankariasacademy.com 53
www.iasparliament.com

3. It is implemented under the aegis of Ministry of


Chemicals and Fertilizers.

Which of the statement(s) giv en above is/are correct?

a. 1 only

b. 2 and 3 only

c. 1 and 2 only

d. 1 , 2 and 3

4) Consider the following statements with respect to


Papua New Guinea
23-05-2023
1. It lies in the southwest Pacific Ocean and
1 ) Panch Karma Sankalp recently seen in news is northeast of Australia.
associated with which of the following sectors?
2. It is situated on the Pacific Ring of Fire.
a. Agriculture
3. Honiara, a key port built with assistance from
b. Tourism China, is located in Papua New Guinea.
c. Biotechnology Which of the statement(s) giv en above is/are correct?
d. Shipping a. 1 only

b. 1 and 2 only
2) Which of the following are members of Forum for c. 1 and 3 only
India-Pacific Islands Cooperation (FIPIC)?
d. 1 , 2 and 3
1. Fiji

2. Philippines
5) Consider the following statements with respect to
3. New Zealand initiativ e on Critical and Emerging Technologies (iCET)
4. Marshall Islands 1. It will be jointly dev eloped by both India and
Japan.
5. Papua New Guinea
2. It helps to remove barriers in technology
Select the correct answer using the codes given below:
transfers and co -production in the areas of
a. 1 and 5 only critical and emerging technologies.

b. 1 , 4 and 5 only Which of the statement(s) giv en above is/are correct?

c. 1 , 3, 4 and 5 only a. 1 only

d. 1 , 2, 3, 4 and 5 b. 2 only

c. Both 1 and 2

3) Consider the following statements with respect to d. Neither 1 nor 2


National Medical Dev ices Policy, 2023

1. It aims to make India a global manufacturing


hub for medical dev ices.

2. Under this the union gov ernment approved


establishment of 1 57 nursing colleges to create
a cadre of skilled professionals.

CHENNAI |SALEM| MADURAI | COIMBATORE DELHI | BANGALORE | T HIRUVANANTHAPURAM


www.shankariasacademy.com 54
www.iasparliament.com

Answers 2. b

1. d The 3rd summit of FIPIC held in Papua New Guinea

Shri Sarbananda Sonowal announces Panch Karma Forum for India-Pacific Islands Cooperation
Sankalp at Chintan Shibir held in Kerala. (FIPIC)

Panch Karm a Sankalp  It was launched during PM Modi's v isit to Fiji


in 201 4.
 Ministry - Ministry of Ports, Shipping &
Waterway s (MoPSW).  Objective – To ex pand India’s strategic and
commercial interests in the pacific regions
 The ‘Panch Karma Sankalp’ includes 5 major through Pacific Island Countries.
announcements
 FIPIC includes 14 of the island countries – Cook
 MoPSW to prov ide 30% financial support for Islands, Fiji, Kiribati, Marshall Islands,
the promotion of Green Shipping. Micronesia, Nauru, Niue, Palau, Papua New
Guinea, Samoa, Solomon Islands, Tonga,
 Under the Green Tug Transition Programme Tuv alu, and V anuatu.
o Jawaharlal Nehru Port,  These countries have large exclusive economic
zones (EEZs) and offer promising possibilities
o V O Chidambaranar Port,
for fruitful cooperation.
o Paradip Port and
3. d
o Deenday al Port,
The Union Cabinet approved the National Medical
o Kandla will procure two tugs each; Devices Policy, 2023.

National Medical Devices Policy, 2023


 Deenday al Port and V O Chidambaranar Port,
Tuticorin to be dev eloped as Green Hydrogen
 Vision – To emerge as the global leader in the
Hub.
manufacturing and innov ation of medical
dev ices by achiev ing 1 0 -1 2% share in the
 Single Window Portal to facilitate and monitor
ex panding global market ov er the next 25
riv er and sea cruises.
y ears.
 Jawaharlal Nehru Port and V O Chidambaranar
 Accelerated growth path with a patient-centric
Port, Tuticorin to become Smart Port by next
approach
y ear.

Green T ug Transition Programme  Policy is ex pected to help the Medical Devices


Sector grow from present $11 Bn to $50 Bn by
 This programme will convert all tugboats into 2030.
Green Hy brid Tugs in India.
 It approv ed establishment of 1 57 nursing
 A tug boat or tugs are marine v essels that colleges to create a cadre of skilled
manoeuvre ships by pushing or pulling them, professionals.
mostly using tow lines.
 Mission
 They tug ships in circumstances where the ships
o Access & Univ ersality
cannot or does not mov e using its own power
like in narrow harbours, canals, etc.
o Affordability
 Green Hy brid Tugs will be powered by Green o Quality
Hy brid Propulsion systems.
o Patient centered & Quality Care
 These Green hy brid tugs will subsequently
adopt non-fossil fuel solutions like (Methanol, o Prev entive & Promotive Health
Ammonia, Hy drogen).
o Security
 The initial target was to start working of green
tugs in all major ports by 2025. o Research and Innovation

 Atleast 50% of all the tugs are likely to be


conv erted into Green tugs by 2030.

CHENNAI |SALEM| MADURAI | COIMBATORE DELHI | BANGALORE | T HIRUVANANTHAPURAM


www.shankariasacademy.com 55
www.iasparliament.com

Strategies to Prom ote Medical Device Sector

 Medical dev ices sector will be facilitated and


guided through a set of strategies that will be
cov er 6 broad areas of policy interventions:

 Regulatory Stream lining - The policy


enhances the role of the Bureau of Indian
Standards (BIS) along with designing a
coherent pricing regulation.

 It will also create a single window clearance


sy stem for licensing of medical devices.
5. b
 Enabling Infrastructure - The
establishment of large medical dev ice parks Launch of INDUS-X to promote partnerships between
under the National Industrial Corridor India and U.S. is scheduled in Washington ahead of
Program and National Logistics Policy 2021 PM’s visit
under the ambit of PM Gati Shakti.
iCET
 Facilitating R&D and Innovation - It aims
at establishing Centres of Ex cellence in  The US-India initiativ e on Critical and
academic and research institutions, innovation Emerging Technologies (iCET) is a platform to
hubs, ‘plug and play ’ infrastructures. accelerate strategic conv ergence and policy
alignment.
 Attracting Inv estments in the Sector -
Along with ex isting schemes it encourages  iCET aims to elev ate and ex pand the strategic
priv ate inv estments, series of funding from technology partnership and defence industrial
V enture Capitalists, Public-Private Partnership cooperation between the 2 c ountries.
(PPP) are planned.
 This initiativ e was unv eiled by President Joe
 Hum an Resources Dev elopment - For Biden and Prime Minister Narendra Modi
skilling, reskilling and upskilling of during the 2022 Quad Leaders' Summit in
professionals in the medical device sector with Toky o.
Ministry of Skill Dev elopment and
Entrepreneurship.  The iCET will be spearheaded by the National
Security Council of both the United States and
 Brand Positioning and Awareness India.
Creation - The policy envisages the creation of
a dedicated Ex port Promotion Council for the  The iCET is ex pected to remove barriers in
medical sector. technology transfers and co -production in the
areas of critical and emerging technologies
4. b
INDUS-X
Papua New Guinea
 It will be launched under the Initiative on
 Papua New Guinea lies in the southwest Pacific, Critical and Emerging Technologies (iCET).
just south of the equator and northeast of
Australia.  It will promote partnerships between two
countries defence innovation ecosystem.
 The country is situated on the Pacific Ring of
Fire, at the point of collision of sev eral tectonic  It focused on adv ancing high-tech cooperation.
plates.

 It is prone to v olcanic activity, earthquakes and


tidal wav es. 24-05-2023
 Most of the terrain is cov ered by tropical rain 1 ) Consider the following statements with respect to
Ordinance
forest.
1. Ordinance making power of president is
mentioned in article 1 23 and it is not subjected
to Judicial Rev iew.

2. President can make ordinances even when both


the houses are not in the session.

CHENNAI |SALEM| MADURAI | COIMBATORE DELHI | BANGALORE | T HIRUVANANTHAPURAM


www.shankariasacademy.com 56
www.iasparliament.com

3. If an Ordinance lapses, the only option for the Which of the statement(s) giv en above is/are correct?
gov ernment is to reissue or re-promulgate it.
a. 1 only
Which of the statement(s) giv en above is/are correct?
b. 2 and 3 only
a. 1 only
c. 1 and 2 only
b. 2 and 3 only
d. 1 , 2 and 3
c. 1 and 2 only

d. 1 , 2 and 3
5) Consider the following statements with respect to
Compressed Bio Gas (CBG) and Compressed Natural
Gas (CNG)
2) It is a scenario when litigants or lawyers attempt to
deliberately move their case to a particular judge or 1. Both CNG and CBG are compressed form of
Court where they think the judgment could be more methane and has the same calorific v alue.
fav orable and this scenario is coined by a particular
term. Which of the following is the respective term used 2. Biogas is produced through the decomposition
for that? of biomass and CNG is a by -product of
Petroleum.
a. Jurisprudence
Which of the statement(s) giv en above is/are correct?
b. Forum Shopping
a. 1 only
c. Grand jury
b. 2 only
d. Probation
c. Both 1 and 2

d. Neither 1 nor 2
3) Consider the following statements with respect to
Hy sterectomy

1. Hy sterectomy is the procedure to remov e the Answers


ov aries and oophorectomy is the remov al of
uterus. 1. b

2. Women can no longer be able to get pregnant The central government promulgated an Ordinance in
after hy sterectomy or get periods regardless of the National Capital Territory (NCT) case.
age.
Ordinance
Which of the statement(s) giv en above is/are correct?
 Article 1 23 of the Constitution defines the
a. 1 only Power of President to promulgate Ordinances
during recess of Parliament.
b. 2 only
 President can promulgate an ordinance only
c. Both 1 and 2 when both the Houses of Parliament are not in
session or when either of the two Houses of
d. Neither 1 nor 2 Parliament is not in session.

 An ordinance made when both the Houses are


in session is v oid.
4) Consider the following statements with respect to
Leatherback Turtles  The power of the President to legislate by
ordinance is not a parallel power of legislation
1. They are found in ev ery ocean except the Arctic and it is subjected to judicial review.
and Antarctic.
 An ordinance can be issued only on those
2. These are the only species of sea turtle that lack subjects on which the Parliament can make
scales and a hard shell. laws.
3. It is listed as Endangered in IUCN’s re d data list
 An Ordinance shall hav e the same force and
and Cites appendix I. effect as an Act of Parliament but the

CHENNAI |SALEM| MADURAI | COIMBATORE DELHI | BANGALORE | T HIRUVANANTHAPURAM


www.shankariasacademy.com 57
www.iasparliament.com

gov ernment is required to bring an Ordinance  It may also inv olve removal of the cervix,
before Parliament for ratification. ov aries, Fallopian tubes, and other surrounding
structures.
 Failure to do so will lead to its lapsing at the
ex piration of 6 weeks from the reassembly of  Hy sterectomies are carried out to treat health
Parliament. problems that affect the female reproductive
sy stem.
Reprom ulgation of Ordinance
 These include:
 The Ordinance may lapse earlier if the
President withdraws it or if both Houses pass o Heav y periods
resolutions disapproving it.
o Long-term pelvic pain
 If an Ordinance lapses, the only option for the
gov ernment is to reissue or repromulgate it. o Non-cancerous tumours (fibroids)

2. b o Ov arian cancer, womb cancer, cervical


cancer or cancer of the fallopian tubes.
CJI Chandrachud condemns forum shopping.
T y pes of hy sterectomy
Forum Shopping of Lawyers
 T otal hysterectomy – The womb and cervix
 It is a practice where litigants file their legal (neck of the womb) are removed.
case in a court which they believe is probable of
prov iding a fav orable v erdict.  Subtotal hysterectomy – The main body of
the womb is removed, leav ing the cerv ix in
 It will happen when litigants or lawyers attempt place.
to deliberately mov e their case to a particular
judge or Court where they think the judgment  T otal hysterectomy with bilateral
could be more fav orable. salpingo-oophorectomy – the womb,
cerv ix, fallopian tubes (salpingectomy) and
 For ex ample, one could directly approach the ov aries (oophorectomy) are removed.
Supreme Court v ia a public interest litigation
case instead of the concerned High Court.  Radical hy sterectomy – The womb and
surrounding tissues are removed, including the
 Forum Non Conv eniens – It refers to a fallopian tubes, part of the v agina, ov aries,
court's discretionary power to decline to ly mph glands and fatty tissue.
ex ercise its jurisdiction where another court, or
forum, may more conveniently hear a case.  Partial hy sterectomies allow for hormone
regulation while total hysterectomies do not.
 Using this power, the court can dismiss a case
in the interests of justice and the parties while 4. c
allocating it to the appropriate bench.
Leatherback Turtles
3. b
 Leatherback turtles are named for their shell,
A recent study shows that after caesarean deliveries, which is leather-like rather than hard, like other
hysterectomies are the 2nd -most frequent procedure turtles.
in w omen of the reproductive age group.
 Habitat - They are the largest sea turtle species
Hy sterectomy and also one of the most migratory, crossing
both the Atlantic and Pacific Oceans.
 It is a surgical procedure to remove the womb
(uterus).  Pacific leatherbacks migrate from nesting
beaches in the Coral Triangle all the way to the
 Oophorectomy is the remov al of ov arie s (the California coast to feed on the abundant
primary source of estrogen). jelly fish.

 Women can no longer be able to get pregnant  Status - IUCN is listed as V ulnerable, but
after the operation. many subpopulations (such as in the Pacific
and Southwest Atlantic) are Critically
 If women hav e not already gone through the Endangered.
menopause, they’ll no longer hav e periods,
regardless of the age.  It is also listed in Cites Appendix I.

CHENNAI |SALEM| MADURAI | COIMBATORE DELHI | BANGALORE | T HIRUVANANTHAPURAM


www.shankariasacademy.com 58
www.iasparliament.com

 The Pacific leatherback turtle populations are a. 1 and 2 only


most at-risk of ex tinction.
b. 2 only
 Conservation Efforts - Pacific leatherbacks
are one of 9 ESA -listed species identified in c. 1 and 3 only
National Oceanic and Atmospheric
Administration (NOAA) Species in the d. 1 , 2 and 3
Spotlight initiative.

5. c
2) Consider the following statements with respect to
 CNG and CBG are both methane based gases
and has the same calorific value. Mohenjo-Daro’s Dancing Girl

 This makes CBG a commercially v iable option


1. It was sculpted during the Indus V alley
as it can be directly used to replace CNG in
Civ ilization (IVC).
transportation fuel.
2. Mohenjo-Daro is located in the present day
 Just like CNG, CBG too can be transported Sindh Prov ince of Pakistan.
through cylinders or pipelines to retail outlets.
3. It was made of iron using the lost wax casting
 The biogas is turned into a gas that is primarily technique.
methane (more than 90%) through
purification, which inv olves removing carbon Which of the statement(s) giv en above is/are correct?
diox ide, hy drogen sulphide and moisture.
a. 1 and 2 only
 This gas is further compressed to high pressure
(250 bar) to produce CBG. b. 2 only

 CBG can be seamlessly utilised in CNG- c. 1 and 3 only


powered v ehicles without requiring any
modifications. d. 1 , 2 and 3

Com pressed Natural


Com pressed Bio Gas (CBG)
Gas (CNG) 3) Consider the following statements with respect to
Sengol
 It can be produced from
any biomass, be it crop 1. It is a historical symbol of Independence as it
residue, cattle dung, signifies the transfer of power from the British
 It is a by -product
sugarcane press mud, to the Indians.
of petroleum
municipal wet waste or
effluents from a sewage 2. It has a Nandi bull on top, sy mbolising justice.
treatment plant.
Which of the statement(s) giv en above is/are correct?

a. 1 only
25-05-2023 b. 2 only
1 ) Consider the following statements with respect to AI
Research, Analy tics and knowledge Assimilation c. Both 1 and 2
(AIRAWAT)
d. Neither 1 nor 2
1. It is a cloud computing platform for Big Data
analy tics with adv anced AI processing
capabilities.
4) Consider the following statements with respect to
2. It is based on the recommendations of the Reusable Launch V ehicle (RLV )
National Strategy for Artificial Intelligence
(NSAI). 1. RLV has 4 stages like PSLV s to propel the
v ehicle into orbit.
3. It works under the aegis of Ministry of Science
2. It uses conventional solid booster designed for
and Technology.
low burn rate.
Which of the statement(s) giv en above is/are correct?
3. ISRO doesn’t launch any reusable launch
v ehicles y et.

CHENNAI |SALEM| MADURAI | COIMBATORE DELHI | BANGALORE | T HIRUVANANTHAPURAM


www.shankariasacademy.com 59
www.iasparliament.com

Which of the statement(s) giv en above  Ministry - Ministry of Electronics and


is/are not correct? Information Technology (MeitY)

a. 1 only  The AIRAWAT PoC of 200 AI Petaflops


integrated with PARAM Siddhi, AI of 210 AI
b. 1 and 3 only Petaflops giv es a total peak compute of 410 AI
Petaflops Mix ed Precision.
c. 2 and 3 only
 It has sustained compute capacity of 8.5
d. 1 , 2 and 3
Petaflops (Rmax ) Double Precision.

 The peak compute capacity (Double Precision,


5) Match the following sports that involves animals with Rpeak) is 1 3 Petaflops.
their respective states
Supercomputers
1. Jallikattu - Tamil Nadu
 Supercomputers aggregate the computational
2. Kambala – Assam power of multiple serv ers or compute devices
for processing huge amounts of data and
3. Dhirio bull fight – Goa performing complex calculations.

4. Bulbul fight – Karnataka Peta flops - A unit of computing speed equal to 1000
million million (1 015) floating-point operations per
Select the correct answer using the codes given below second.

a. 1 , 2 and 3 only 2. a

b. 2 and 4 only PM Modi unveiled the famous Dancing Girl of


Mohenjodaro at the International Museum Expo in
c. 1 and 3 only Delhi’s Pragati Maidan.

d. 1 , 2, 3 and 4 Dancing Girl

 It is a bronze statuette created over 4,500 years


ago during the Indus v alley Civ ilisation.
Answers
 It was found in the ancient Mohenjo-Daro site
1. a
in 1 926.
AI supercomputer 'AIRAWAT' has secured the 75th
position in the prestigious Top 500 Global  This statue reflecting the aesthetics of a female
Supercomputing List. body as conceptualized during that historical
period.
AIRAWAT (AI Research, Analy tics and
knowledge Assimilation)  The bronze girl was made using the lost-wax
casting technique.
 It is an AI-based cloud computing
infrastructure (supercomputer) based on the  The statuette was named Dancing Girl based on
recommendations of the National Strategy for an assumption of her profession.
Artificial Intelligence (NSAI).
 She is one of two bronze artworks found at
 This platform will assist in the R&D of new Mohenjo-Daro that shows a more natural pose
technologies that help in solv ing business and than compared to other more formal figures.
gov ernance use cases.
 The statuette has large eyes, a flat nose, healthy
 The installation of AIRAWAT is at C-DAC Pune. cheeks, curly hair, and a broad forehead.

 She is a tall figure with long legs and arms, high


neck, subdued belly, and sensuously modeled.

 The girl wears some bangles and a necklace.

 She has 25 bracelets on her left arm and four


bangles on her right arm and is holding an
object in her left hand.

CHENNAI |SALEM| MADURAI | COIMBATORE DELHI | BANGALORE | T HIRUVANANTHAPURAM


www.shankariasacademy.com 60
www.iasparliament.com

 Her long hair sty led as big bun rested on her 4. b


shoulder.
A SpaceX Falcon 9 rocket carrying a Qatari
Lost Wax T echnique communications satellite launches from the Kennedy
Space Center in Florida.
 This wax model is then covered with a clay
coating, leaving some holes as passageways. Reusable Launch Vehicle (RLV)

 When the clay -covered moulds are heated in  The RLV has only two stages to propel the
the ov en, the wax melts out. v ehicle into orbit.

 After the mould has cooled down, the outer clay  Once the fuel in the first stage has been
cov er is chipped off and the finishing touches ex pended, the v ehicle will shed it, and carry on
are done to the solid bronze statue. with the second stage.

3. c  Once it has been shed, the first stage will re -


enter the atmosphere and land in an
Sengol was kept in Allahabad museum will be installed autonomous fashion at a pre -determined
in the new Parliament building by Prime Minister location.
Narendra Modi.
 After some maintenance, it will be av ailable for
Historical significance reuse.

 Tamil kings had this sengol (a Tamil word for  The configuration of RLV -TD is similar to that
sceptre), which is a sy mbol of justice and good of an aircraft and combines the complexity of
gov ernance. both launch v ehicles and aircraft.

 The two great epics Silapathikaram and  The winged RLV -TD has been configured to act
Manimekalai records the significance of a as a fly ing test bed to ev aluate v arious
sengol. technologies, namely , hy personic flight,
autonomous landing and powered cruise flight.
 A ceremony performed during the Chola
dy nasty, in which the transfer of power from  In future, this v ehicle will be scaled up to
one king to the other was sanctified and blessed become the first stage of India’s reusable two
by Samay acharyas (spiritual leaders). stage orbital launch v ehicle.

 It also has historical significance of  RLV -TD consists of a fuselage (bo dy ), a nose
Independence as it signifies the transfer of cap, double delta wings and twin v ertical tails.
power from the British (Lord Mountbatten, the
last British v iceroy), to the Indians (Nehru).  It also features sy mmetrically placed active
control surfaces called Elevons and Rudder.
 Nehru and Rajaji reached out to
Thiruv aduthurai Atheenam, a well-known mutt  This technology demonstrator was boosted to
in Tanjore district, Tamilnadu for the Mach no: 5 by a conv entional solid booster
manufacturing of sengol. (HS9) designed for low burn rate.

Sengol  The selection of materials like special alloys,


composites and insulation materials for
 It deriv es from the Tamil word semmai, dev eloping an RLV -TD and the crafting of its
meaning righteousness. parts is v ery complex and demands highly
skilled manpower.
 The sceptre measures 5 feet in length and has a
Nandi bull on top, sy mbolising justice.  Many high technology machinery and test
equipment were utilised for building this
 This is a gold coated silver scepter. v ehicle.

 The ceremony added the song played during the India


ceremony was composed by the 7 th-century
Tamil saint Tirugnana Sambandar.  In 201 0, ISRO began dev eloping a winged
reusable rocket, taking the first step towards
 The Sengol was kept in Allahabad museum. realising a two-stage-to-orbit (TSTO) launch
v ehicle that could be fully reusable.

 On 201 6, the winged vehicle successfully flew at


hy personic speed.

CHENNAI |SALEM| MADURAI | COIMBATORE DELHI | BANGALORE | T HIRUVANANTHAPURAM


www.shankariasacademy.com 61
www.iasparliament.com

5. c 26-05-2023
Jallikattu, the traditional rural sport involving bulls, 1 ) Consider the following statements with respect to
has received judicial approval. ANZUS

Jallikattu 1. It is a military alliance with Australia, US and


New Zealand as founding members.
 The term Jallikattu is a union of two words -
‘Calli’ (coins) and ‘Kattu’ (tie), which denotes a 2. It is signed in 1 951 to protect the security of the
bundle of coins tied to the bull’s horns. Pacific region.

 Jallikattu is a 2,000 years old competitive bull Which of the statement(s) giv en above is/are correct?
taming sport in Tamil Nadu which contestants
attempt to tame a bull for a prize, wherein if a. 1 only
they fail, the bull owner wins the prize.
b. 2 only
 It is celebrated in the second week of January at c. Both 1 and 2
the time of the Pongal (harv est) festival.
d. Neither 1 nor 2
 Kangay am, Pulikulam, Umbalachery, Bargur
and Malai Maadu are among some nativ e cattle
breeds reared for Jallikattu.
2) The term Meiogyne Arunachalensis recently seen in
 Its first references date back to a seal discovered news is related to?
at Mohenjodaro, which is dated between 2,500
BC and 1 ,800 BC. a. It is a newly discovered tree species in Arunachal
Pradesh.
 Back then, it was called Eru Thazuv al, meaning
embracing the bull. b. It is a hornbill species that goes ex tinct in
Arunachal Pradesh.
 Jallikattu also finds mention in
Silappadikaram, one of the great Tamil epics c. It is a rare species of mushroom only found in
from the Sangam age. Arunachal Pradesh.

 The best guide to the cultural univ erse of d. It is a medicinal plant in Arunachal Pradesh that
Jallikattu is C S Chellappa’s brilliant nov ella, increases metabolism in humans.
‘V aadiv asal’ (Arena), written in the 1 940s.

Sports inv olving Animals


3) Consider the following statements with respect to
 Rekla – Tamil Nadu Amazon Face (FACE) Project

1. It is a field ex periment that ex poses tropical


 Bail Gadi Shariat – Maharashtra
trees to sequester future CO2 concentrations in
 Kambala – Karnataka the western ghats region.

2. It is led by the Union Env ironment Ministry in


 Maramudi Bull Surfing – Kerala
partnership with Amazon.
 Dhirio bull fight – Goa Which of the statement(s) giv en above is/are correct?
 Buffalo fight during Bhogali Bihu festiv al - a. 1 only
Assam
b. 2 only
 Camel racing – Rajasthan
c. Both 1 and 2
 Bulbul fight - Assam
d. Neither 1 nor 2
 Cock fighting - Andhra Pradesh, Chattisgarh,
Jharkhand Kambala – Karnataka

CHENNAI |SALEM| MADURAI | COIMBATORE DELHI | BANGALORE | T HIRUVANANTHAPURAM


www.shankariasacademy.com 62
www.iasparliament.com

4) Match the following  The members met annually to discuss their


shared interests and concerns.
Elem ents of Mandala in art & their
Meaning  In 1 984, New Zealand dec lared its country a
nuclear-free zone and refused to allow U.S.
1. The eight spokes of the wheel - Eight-fold nuclear-powered submarines to v isit its ports.
path of Buddhism
 On 1 986, the United States suspended its treaty
2. The lotus flower - Peace obligations toward New Zealand and reduced
the two countries military ties.
3. The sun - Action and
energy  The 3 nations remained formal parties to the
treaty, but ANZUS was inoperative.
4. Triangles facing upward - Univ erse
2. a
Select the correct code:
Meiogy ne Arunachalensis
a. Only one is correct

b. Only two pairs are correct  Genus - Meiogy ne

c. Only three pairs are correct  It is distributed throughout South and


Southeast Asia and includes approximately 33
d. All the pairs are correct described taxa.

 Habitat - It is the first record of the genus from


Northeast India and the eastern Himalayan
5) Which of the following statements is/are correct biodiv ersity hotspot.
about Gini coefficient
 It is also found in interv ening districts of Lower
1. It is a measure of the distribution of income Dibang V alley and Lohit, and also in northern
across a population, it ranges from 0 to parts of My anmar in areas neighbouring the
Namdapha National Park.
2. It is usually defined mathematically based on
the Laffer Curv e, which plots the proportion of  It is the largest species of the genus described
the total income of the population. so far in terms of tree height and girth of the
tree trunk.
Select the correct answers using the c odes giv en below
 The species has morphological similarity with
a. 1 only Meiogy ne max iflora, a species distributed in
Thailand, but it differs in a number of
b. 2 only
v egetative and reproductive characteristics.
c. Both 1 and 2
3. d
d. Neither 1 nor 2
Free Air CO2 Enrichment (FACE)

 It is a field ex periment that ex poses tropical


trees to sequester future CO2 concentrations in
Answers Amazon forest, Brazil.
1. c
 It is led by the National Institute for Amazon
ANZUS Research (INPA) along with a range of partners.

 Countries - Australia, New Zealand, US  It is to increase the functioning of the world's


(ANZUS) largest tropical forest in light of climate change
and to apply the knowledge gained for steering
 It was a trilateral defence agreement signed in regional policies on climate change mitigation
1 951 to protect the security of the Pacific. and adaptation.

 It was an arrangement made after the Second  It hav e been used successfully in sev eral
World War and the early stages of the Cold War. locations including

 It was further ratified by the United States and o Duke Forest,


entered into force in 1 952.
o North Carolina,

CHENNAI |SALEM| MADURAI | COIMBATORE DELHI | BANGALORE | T HIRUVANANTHAPURAM


www.shankariasacademy.com 63
www.iasparliament.com

o USA,  A higher Gini index indicates greater inequality,


with high income indiv iduals receiving much
o Staffordshire, larger percentages of the total income of the
population.
o UK (BIFoR FACE) and New South
Wales,  The Gini coefficient is usually defined
mathematically based on the Lorenz curve,
o Australia (EucFACE). which plots the proportion of the total income
of the population (y -axis) that is cumulatively
4. a
earned by the bottom x % of the population.
Mandala in art

 Mandala patterns are a centuries-old motif that


are used to depict the cosmos, and hav e been 30-05-2023
adapted by artists the world over, each of whom 1 ) Consider the following statements with respect to
hav e added their own interpretation and NV S-01 Mission
painted it as their own.
1. It is the first of the 2nd generation satellites
 Literally meaning “circle” or “centre” in env isaged for the Nav igation with Indian
Sanskrit, mandala is defined by a geometric Constellation (NavIC) services.
configuration that usually incorporates the
circular shape in some form. 2. It was launched using Small Satellite Launch
V ehicle (SSLV ) into a Geosynchronous Transfer
 While it can also be created in the shape of a Orbit.
square, a mandala pattern is essentially
interconnected. It is believ ed to be rooted in 3. For the first time, an indigenous atomic clock
Buddhism, appearing in the first century BC in will be flown in this mission.
India.
Which of the statements giv en above is/are correct?
 There are v arious elements incorporated within
the mandala, each of which has its own a. 1 only
meaning.
b. 1 and 3 only
 For instance, the eight spokes of the wheel (the
c. 2 and 3 only
dharmachakra) represent the eightfold path of
Buddhism (practices that lead to liberation d. 1 , 2 and 3
from rebirth).

 The lotus flower depicts balance, and the sun


represents the universe. 2) Consider the following statements with respect to
Intergovernmental Negotiating Committee on Plastic
 Facing up, triangles represent action and pollution
energy , and facing down, they represent
creativity and knowledge. 1. It was established to dev elop an international
legally binding instrument on plastic pollution,
5. a including in the marine env ironment.
Gini coefficient 2. It was established in 201 4 during the COP 20
held in Lima, Peru.
 It is often used as a gauge of economic
inequality (income), measuring income Which of the statements giv en above is/are correct?
distribution or, less commonly, wealth
distribution among a population. The a. 1 only
coefficient ranges from 0 (or 0%) to 1 (or
1 00%). b. 2 only

 A coefficient of zero indicates a perfectly equal c. Both 1 and 2


distribution of income or wealth within a
population. d. Neither 1 nor 2

 A coefficient of one represents a perfect


inequality when one person in a population
receives all the income, while other people earn
nothing.

CHENNAI |SALEM| MADURAI | COIMBATORE DELHI | BANGALORE | T HIRUVANANTHAPURAM


www.shankariasacademy.com 64
www.iasparliament.com

3) Consider the following statements with respect to Answers


Joint Child Malnutrition Estimates (JME) 2023
1. b
1. It is jointly released by UNICEF, WHO and
World Bank. ISRO’s GSLV -F1 2 successfully places navigation
satellite NV S-01 into intended orbit.
2. According to this estimate, Prev alence of
ov erweight children decreased in India in 2022 NVS-01 Mission
compared to 201
 It is indigenously designed and dev eloped by
3. According to this estimate, India continues to Indian Space Research Organization (ISRO).
show a reduction in stunted children under 5
y ears in 2022 compared to 2012.  Weight - 2232 kg

Which of the statement(s) giv en above is/are correct?  Position - Geosy nchronous Transfer Orbit.

a. Only one  It is the first of the 2nd generation satellites


env isaged for the Nav igation with Indian
b. Only two Constellation (NavIC) services.

c. All three  For the first time, an indigenous atomic clock


will be flown in NV S-01 .
d. None
 The satellite would prov ide real-time
positioning and timing services over India and
a region approximately 1 ,500 km around the
4) The term Zombie Lending recently seen in the news
mainland.
is related to?
 This mission is the 6th operational flight of the
a. It refers to the practice of providing credit to
GSLV with indigenous cryogenic stage.
entities that do not hav e the capability to repay

b. It refers to the funding of an insolvent NBFC by  Geosy nchronous Satellite Launch


a nationalized bank for their restructuring process Vehicle (GSLV) or GSLV Mk-II is designed to
carry the nav igation satellite.
c. It refers to the practice of lending to a shell
company and div erting its funds  Adv antages – Terrestrial, aerial and maritime
nav igation, precision agriculture, location-
d. It refers to a Foreign Portfolio Investments in the based serv ices in mobile dev ices and marine
budding MSMEs fisheries.

2. a

5) Consider the following statements with respect to A United Nations committee (INC) is meeting on May
Black-winged Stilts 29 in Paris, France to w ork on a landmark treaty to
end global plastic pollution.
1. It is found in wetlands with open shallow water,
often in brackish habitats. Intergovernmental Negotiating Committee for
Plastics (INC)
2. It is listed as v ulnerable in IUCN’s Red list of
threatened species.  In 2022, at the 5th session of the United
Nations Env ironment Assembly (UNEA), a
Which of the statement(s) giv en above is/are correct? resolution was adopted to dev elop an
international legally binding instrument on
a. 1 only plastic pollution, including in the marine
env ironment with the ambition to complete the
b. 2 only negotiations by end of 2024.
c. Both 1 and 2  The Intergovernmental Negotiating Committee
(INC) was charged with dev eloping the treaty
d. Neither 1 nor 2
on plastic pollution.

 The current meeting is the 2nd of fiv e meetings


due to take place to complete the negotiations
by the end of 2024.

CHENNAI |SALEM| MADURAI | COIMBATORE DELHI | BANGALORE | T HIRUVANANTHAPURAM


www.shankariasacademy.com 65
www.iasparliament.com

 First meeting being held in Punta del Este,


Uruguay .

 It starts to begin its work in the 2nd half of 2022


with the ambition of completing its work by the
end of 2024.

 It needs to include marine, land and water


plastic pollution, promote sustainable
production and consumption of plastics
through, among other things.

 IUCN has play ed a key role globally in helping


identify , raise awareness, and provide tools to
assess the impacts and sources of plastic  Stunting - It is when a child has a low height
pollution. for their age.

3. b  Wasting - It is when a child is too thin for their


height.
India sees reduction in stunting but wasting, obesity
are concerns.

Joint Child Malnutrition Estimates (JME) 2023

 It is jointly released by UNICEF, WHO and


World Bank.

Global Scenario

 Obesity - The prevalence of obesity marginally


increased in a decade in 2022 compared to
201 2.

 But the ov erall classification for obesity is low


and much lower than the global prevalence.

 Stunting - It is declined from a prevalence rate


of 26.3% in 201 2 to 22.3% in 2022.

 In South Asia, the decline was much sharper as


it dropped from 40.3% to 30.5%.
T argets for 2030
 Wasting - It is probably more complex in
South Asia and India.

 While in Africa it starts from 4-6 months.

In India

 Obesity - Prev alence of ov erweight children


increased in India in 2022 compared to 2012. 4. a

 Stunting - India continues to show a reduction RBI Governor Shaktikanta Das w arned about the
in stunted children under 5 y ears in 2022 banks adopting innovative methods for evergreening
compared to 2012. of loans.

 Wasting - In India 2/3rds of children at 12 or Zom bie Lending


24 months had wasting at birth or at one month
 Zombie lending refers to the practice of
of age.
prov iding credit to entities that do not have the
 It is caused by maternal malnutrition. capability to repay.

 Ev ergreening - If an account turns into a


non-performing asset (NPA), banks are
required to make higher provisions which will
impact their profitability.

CHENNAI |SALEM| MADURAI | COIMBATORE DELHI | BANGALORE | T HIRUVANANTHAPURAM


www.shankariasacademy.com 66
www.iasparliament.com

 A loan turns into a nonperforming asset, if the


interest or instalment remains unpaid for more
than 90 day s.

 So, to av oid c lassifying a loan as an NPA, banks


adopt the ev er-greening of loans.

 In the past, many banks had indulged in


dressing up bad loans and giv en additional
funds to companies who didn’t hav e the
capacity to repay.

 An increase in lending to unproductive firms,


popularly referred to as zombies will inflates
credit growth and the resultant loan defaults
haunt financial institutions at a later stage.

5. a

Black-winged stilts use platforms rising out of water to


build their nests

Black-winged Stilts

 These black-winged stilt species do not


normally occur in forests.

 It is found in wetlands with open shallow water,


often in brackish habitats.

 The black-winged stilt species are partially


migrant birds.

 IUCN status - Least Concern (LC).

 T he CITES status - Not Ev aluated.

 It is closely related to the black stilt


(Himantopus nov aezelandiae).

 It is distributed in Asia, Africa, Australia,


Europe, North America and South America.

 The black-winged stilt populations in


temperate, sub-tropical and tropical ranges are
usually sedentary and resident.

 They use platforms rising out of water to build


their nests.

CHENNAI |SALEM| MADURAI | COIMBATORE DELHI | BANGALORE | T HIRUVANANTHAPURAM

You might also like